You are on page 1of 34

Join Our Telegram Group - t.

me/pdf4me

VISIONIAS
www.visionias.in
ANSWERS & EXPLANATIONS
GENERAL STUDIES (P) TEST – 2980 (2020)

Q 1.B
• Monotremes are mammals that lay eggs (Prototheria) instead of giving birth to live young like
marsupials (Metatheria) and placental mammals (Eutheria).
• There are only five living monotreme species: the duck-billed platypus and four species of echidna
(also known as spiny anteaters). All of them are found only in Australia and New Guinea.
• Monotremes are the only group of mammals that lay eggs, i.e. they are oviparous, laying one to three
eggs. They have a single posterior opening, the cloaca, for excretion and reproduction. The name
monotreme means one-holed.
• Monotremes resemble other mammals in producing milk to nourish their young, in having three inner ear
bones and a single bone in the lower jaw. Monotremes are highly specialized feeders and the adults have
no teeth.

Q 2.B
• Taiga, also called boreal forest is a biome (major life zone) of vegetation composed primarily of
cone-bearing needle leaved or scale leaved evergreen trees, found in northern circumpolar forested
regions characterized by long winters and moderate to high annual precipitation.
• The tree line is the edge of the habitat at which trees are capable of growing. It is found at high
elevations and high latitudes. Beyond the tree line, trees cannot tolerate the environmental conditions. It
occurs at the northern flank of the taiga ecosystem. Beyond the tree line, tundra ecosystem is
present. Hence statement 1 is not correct.
• Soils in the boreal forest are typically podzols, gray soils that are thin, acidic, and poor in
nutrients. These soils lie beneath a mat of coniferous tree needles and other organic material that
accumulates due to the slow decomposition rates and limited soil microorganism activity that occurs in
the cold climate. Tannins and other acidic compounds from this layer cause the upper layers of soil to
become acidic. Hence statement 2 is correct.
• Boreal forests have lower productivity than tropical or temperate forests; they also have less
diversity, with only a tree layer and ground layer. Temperatures in the arctic tundra are cold year-round
and precipitation is very low. However their primary productivity is much higher than tundra.
Q 3.D
• The ecads otherwise known as ecophenes are genetically similar but morphologically distinct in
response to different environmental conditions. The variation in morphological characters with respect
to leaf shape, number of branches, height of the individuals, and length, number and colour
of inflorescences according to the intensity of light available is common in plant species. In the species,
Plantago lanceolata, the ribwort plantain, populations adapted to either sun or shaded habitats show
large differences in growth form.
• The influence of temperature for the ecad formation is remarkable in plant species. It is reported that
under optimal night temperatures, the flowers are small whereas, under low night temperatures, the
flowers have larger ovaries in Capsicum annum.
• Soil wetness is one of the primary factors for microclimatic variation in a habitat that in turn induces
ecad formations in plant species. For example, the species, Ricinus communis exhibits much
variation according to soil moisture regarding the size and ornamentation of the regma (fruit) and colour
and patterning of the seeds, fruits, leaves pollens.
• The content of metals in the soil is also found to be an influential factor to initiate morphological
variations in some plant species. The differences in leaf dimensions between various mine populations
of B. homblei are noted. Leaves of this species from high copper soils were significantly longer and
narrower than those of plants from low copper soils.
1 www.visionias.in ©Vision IAS

All Exam Study Material Available On - PHOTOCOPYNOTES.COM


Join Our Telegram Group - t.me/pdf4me
Q 4.B
• Byssinosis is a rare lung disease. It’s caused by inhaling hemp, flax, and cotton particles and is
sometimes referred to as brown lung disease. It’s a form of occupational asthma.
• In the United States, byssinosis occurs almost exclusively in people who work with unprocessed cotton.
People who open bales of cotton during the first stage of processing are at the highest risk. There’s also a
type of byssinosis called grain worker’s lung that appears in people who work with grains.
• Patients with byssinosis usually have difficulty with cough and feelings of chest tightness. Some develop
“Monday fever” when they are exposed to the dust as they return to work after a break.
• The symptoms improve over the course of the week, and usually cause no long-term effects if the
exposure is stopped. However, permanent damage and difficulty in breathing can occur with continued
exposure. Most people with symptoms have had exposure for more than 10 years.

Q 5.A
• Recently, India has joined the G20 Global Smart Cities Alliance on Technology Governance. The G20
Global Smart Cities Alliance is a league of 15 of the world's leading city networks and technology
governance organizations that will work towards advancing responsible and ethical use of smart city
technologies.
• Statement 1 is correct: The aim of the alliance is to promote the responsible and ethical use of smart city
technologies by establishing global norms and policy standards for the use of connected devices in public
spaces. Currently, there is no global framework or set of rules in place for how sensor data collected in
public spaces, such as by traffic cameras, is used. The effort aims to foster greater openness and trust as
well as create standards for how this data is collected and used.
• Global Smart Cities Alliance will advance how technology is used in public places and promote core
principles including transparency, privacy and security
• Statement 2 is not correct: The alliance unites municipal, regional and national governments, private-
sector partners and cities’ residents around a shared set of core guiding principles for the implementation
of smart city technologies. The World Economic Forum, the International Organization for Public-Private
Cooperation, has been selected to act as the secretariat for a new G20 Global Smart Cities Alliance.
• Statement 3 is not correct: The Global Smart Cities Alliance's founding set of institutional partners
include the presidents and host nations of the Group of 20 (G20) in 2019 and 2020; Japan and the
Kingdom of Saudi Arabia; the Smart City Mission of India; Cities for All; Cities Today Institute;
Commonwealth Local Government Forum; Commonwealth Sustainable Cities Network among others.

Q 6.A
• Biomagnification refers to the tendency of pollutants to concentrate as they move from one tropic level to
the next. Thus in biomagnification there is an increase in concentration of a pollutant from one link in a
food chain to another.
• In order to bimagnification to occur, the pollutant must be long lived, mobile and soluble in fats. If
pollutant is short lived, it will be broken down before it can become dangerous.
• If it is not mobile, it will stay in one place and unlikely to be taken up by the organism. Pollutants that
dissolve in fats, however, may be retained for a long time.
• It is traditional to measure the amount of pollutants in fatty tissues of organism such as fish. In mammals,
we often test the milk produced by females, since the milk has a lot of fat in it are often more susceptible
to damage from toxins (poisons).

Q 7.B
• Pyrolysis and gasification are two important processes that are used to decompose materials. Both these
processes are different from combustion because the combustion is carried out in the presence of an
excessive amount of oxygen.
• What is Pyrolysis?
o Pyrolysis is the process of thermal conversion of organic matter using a catalyst in the absence of
oxygen or near absence. Therefore, it is the decomposition of material in an inert atmosphere. It is a
chemical reaction that includes alteration of the chemical composition of the material. Moreover, it is
a reversible process.
o In pyrolysis, what we do is heating a material to a temperature above its decomposition temperature.
It breaks down the chemical bonds of the material.
o Therefore, this process usually forms small molecules from large fragments. But, these small
molecules can combine, forming large molecular masses as well. For example, pyrolysis of
triglycerides form alkanes, alkenes, alkadienes, aromatics and carboxylic acids.
2 www.visionias.in ©Vision IAS

All Exam Study Material Available On - PHOTOCOPYNOTES.COM


Join Our Telegram Group - t.me/pdf4me
o Moreover, the process proceeds at temperatures ranging from 350°C – 600°C
• What is Gasification?
o Gasification is a thermo-chemical process that converts biomass into a combustible gas called
producer gas (syngas). Here, the materials decompose in an environment where a little amount of
oxygen is present. However, this amount of oxygen is not enough for combustion. The products of
gasification are heat and combustible gas. The process proceeds at temperatures ranging from 800°C
– 1200°C.
o The principle components in the combustible gas that forms during this process include carbon
monoxide and hydrogen gas. In addition, there are some other components such as water vapour,
carbon dioxide, tar vapour, ash, etc.
o Moreover, pyrolysis is useful for applications in food manufacturing, i.e. caramelization, production
of fuel from biomass, production of ethylene, to treat plastic waste, etc. while gasification is useful for
heat production, production of electricity, etc.
• Hence only statement 2 is correct.

Q 8.B
• The Pradhan Mantri Innovative Learning Programme: DHRUV has been started by Ministry of Human
Resource Development, Government of India to identify and encourage talented children to enrich
their skills and knowledge. In centres of excellence across the country, gifted children will be mentored
and nurtured by renowned experts in different areas, so that they can reach their full potential.
• The first batch of DHRUV programme was implemented during October 2019.
• 60 outstandingly talented students were selected in the first batch of DHRUV programme. To begin with,
two areas i.e. Science and Performing Arts were covered. There were 60 students in all, 30 from each
area. The 60 students came from across the country. The students have been broadly chosen from classes
9 to 12, from all schools including government and private.

Q 9.D
• Dioxin and furan are chlorinated aromatic compounds having tremendous toxic effects, carcinogenicity
and persistence in the environment; hence these Persistent Organic Pollutants (POPs) are real threats to
the environment and existence of mankind.
• These compounds do not occur naturally, nor they are produced intentionally. In fact these are the
byproducts of industrial and pollution control related operations, barring few catastrophic or accidental
origins like volcanoes, forest fires and accidental fires etc.
• Sources:
o Municipal Solid Waste Incinerator,
o Open burning of domestic waste,
o Bio-Medical waste incinerators and hazardous waste incinerators,
o Industrial boilers and furnaces,
o Petroleum refining,
o Biogas combustion and landfill gas combustion,
o Candles, Crematoriums, Cigarette smoking,
o PVC manufacturing,
o Dyes and pigments,
o Motor vehicle fuel combustion (Diesel and gasoline), Coal combustion for industrial, residential
and commercial purposes.
• Hence all the options are correct.

Q 10.D
• Algae is commercially cultivated for Pharmaceuticals, Nutraceuticals, Cosmetics and Aquaculture
purpose. Humans use algae as food, for production of useful compounds, as biofilters to remove nutrients
and other pollutants from wastewaters, to assay water quality, as indicators of environmental change, in
space technology, and as laboratory research systems.
• Carbon Dioxide Fixation: Like any other plant, algae, when grown using sunlight, consume (or absorb)
carbon dioxide (CO2) as they grow, releasing oxygen (O2). For high productivity, algae require more
CO2, which can be supplied by emissions sources such as power plants, ethanol facilities, and other
sources.
• Bio-fuel & Oil extraction: Algae can be used to make Biodiesel (see algaculture), Bioethanol and
biobutanol and can produce vastly superior amounts of vegetable oil, compared to terrestrial crops grown
for the same purpose.
3 www.visionias.in ©Vision IAS

All Exam Study Material Available On - PHOTOCOPYNOTES.COM


Join Our Telegram Group - t.me/pdf4me
• Purification of wastewater: Algae thrive in nutrient-rich waters like municipal waste waters (sewage),
animal wastes and some industrial effluents, at the same time purifying these wastes while producing a
biomass suitable for biofuels production.
• Food Supplements: Algae are national foods of many nations: China consumes more than 70 species,
including fat choy, a cyanobacterium considered a vegetable; Japan, over 20 species; Ireland, dulse; Chile,
cochayuyo. Laver is used to make "laver bread" in Wales where it is known as bara lawr; in Korea, gim;
in Japan, nori and aonori. It is also used along the west coast of North America from California to British
Columbia, in Hawaii and by the Māori of New Zealand.

Q 11.B
Recent context- India’s first coal gasification based fertiliser plant to be set up in Talcher, Odisha.
Coal gasification
• It is one of the clean coal technologies and involves the process of converting coal into synthesis gas (also
called syngas).
Syngas
• It is an abbreviation for synthesis gas, which is a mixture comprising carbon monoxide, carbon dioxide,
and hydrogen. Hence statement 2 is not correct.
• The syngas is produced by gasification of a carbon-containing fuel to a gaseous product that has some
heating value. Some of the examples of syngas production include gasification of coal emissions, waste
emissions to energy gasification, and steam reforming of coke.
• The by-products of coal gasification include coke, coal tar, sulfur, ammonia and fly ash, all having their
own potential uses. CO2 and ammonia are further reacted to produce urea.
• Syngas can also be used in a variety of other applications such as in the production of electricity, fuel for
IC engines, making plastics, cement etc. It can also be an intermediate in the industrial synthesis of
ammonia and fertilizer. Hence statement 3 is correct.
• The move towards the reduction of greenhouse gas emissions with the use of syngas as a fuel for
the production of electricity is predicted to grow sharply in the near future as a result of:
o global warming and ocean surface acidification, which imply the need to control and reduce
greenhouse gas emission,
o the low estimated reserves of oil, and rising cost as well as demand for natural gas and the need to
reduce the dependence on fossil fuel imports from unstable areas. Hence statement 1 is correct
Process of coal gasification
• The general raw materials used for gasification (creation of syngas) are coal, petroleum-based materials,
or other materials that would be rejected as waste. From these materials, a feedstock is prepared. This is
inserted into the gasifier in dry or slurry form. In the gasifier, this feedstock reacts in an oxygen-starved
environment with steam at elevated pressure and temperature. The resultant syngas is composed of 85%
carbon monoxide and hydrogen and small amounts of methane and carbon dioxide.

Q 12.A
• The Cabinet Committee on Economic Affairs, chaired by Hon'ble Prime Minister Shri Narendra Modi has
approved the "Pradhan Mantri JI-VAN (Jaiv Indhan- Vatavaran Anukool fasal awashesh Nivaran)
Yojana" for providing financial support to Integrated Bioethanol Projects using lignocellulosic
biomass and other renewable feedstock.
Financial Implications:
• The JI-VAN Yojana will be supported with total financial outlay of Rs.1969.50 crore for the period from
2018-19 to 2023-24. Out of scheme fund of Rs.1969.50 crore, Rs.1800 crore has been allocated for
supporting 12 Commercial projects, Rs.150 crore has been allocated for supporting 10 demonstration
Projects and remaining Rs.9.50 crore will be provided to Centre for High Technology (CHT) as
administrative charges.
Details of the scheme
• Under this Yojana, 12 Commercial Scale and 10 demonstration scale Second Generation (2G) ethanol
Projects will be provided a Viability Gap Funding (VGF) support in two phases:
a) Phase-I (2018-19 to 2022-23): wherein six commercial projects and five demonstration projects will
be supported.

4 www.visionias.in ©Vision IAS

All Exam Study Material Available On - PHOTOCOPYNOTES.COM


Join Our Telegram Group - t.me/pdf4me
b) Phase-II (2020-21 to 2023-24): wherein remaining six commercial projects and five demonstration
projects will be supported.
• The scheme focuses to incentivise 2G Ethanol sector and support this nascent industry by creating a
suitable ecosystem for setting up commercial projects and increasing Research & Development in this
area.
Other objectives of the scheme
• Meeting Government of India vision of reducing import dependence by way of substituting fossil fuels
with Biofuels
• Achieving the GHG emissions reduction targets through progressive blending/ substitution of fossil fuels.
• Addressing environment concerns caused due to burning of biomass/ crop residues & improve health of
citizens.
• Improving farmer income by providing them remunerative income for their otherwise waste agriculture
residues.
• Creating rural & urban employment opportunities in 2G Ethanol projects and Biomass supply chain
• Contributing to Swacch Bharat Mission by supporting the aggregation of nonfood biofuel feedstocks such
as waste biomass and urban waste
• Indigenizing of Second Generation Biomass to Ethanol technologies
• The ethanol produced by the scheme beneficiaries will be mandatorily supplied to Oil Marketing
Companies (OMCs) to further enhance the blending percentage under EBP Programme.

Q 13.A
• Fossils were recently unearthed in southern Germany of a remarkable ape that lived about 11.6 million
years ago may dramatically alter the understanding of the evolutionary origins of a fundamental human
trait - walking upright on two legs. Hence option (a) is the correct answer.
• Scientists said the ape, called Danuvius guggenmosi, combined attributes of humans - straight lower
limbs adapted for bipedalism - with those of apes - long arms able to stretch out to grasp tree branches.
That indicates Danuvius was able to walk upright on two legs and also use all four limbs while clambering
through trees.
• It is the oldest-known example of upright walking in apes. The discovery suggests that bipedalism
originated in a common ancestor of humans and the great apes - a group that includes chimpanzees,
bonobos, gorillas and orangutans - that inhabited Europe rather than an ancestor from Africa, the
continent where our species Homo sapiens first appeared roughly 300,000 years ago.
• Until now, the oldest fossil evidence of bipedalism in humankind’s evolutionary tree dated to about 6
million years ago: fossils from Kenya of an extinct member of the human lineage called Orrorin
tugenensis as well as footprints on the Mediterranean island of Crete. If Danuvius turns out to be ancestral
to humans, that would mean that some of its descendants at some point made their way to Africa.

Q 14.C
• Extended producer responsibility, a practice and a policy approach in which producers take
responsibility for management of the disposal of products they produce once those products are
designated as no longer useful by consumers. Responsibility for disposal may be fiscal, physical, or a
combination of the two.
• Motivations for extended producer responsibility practices include a mixture of economic, environmental,
and social factors. Extended producer responsibility shifts the economic burden of the cost of
disposal from the government to the producer of the product. Within an environmental context,
products must be designed for recyclability, and extended producer responsibility encourages design for
recycling while discouraging the use of toxic components in the product. Finally, extended producer
responsibility meets increasing consumer demand for environmentally friendly products that can easily be
recycled or are manufactured using recycled content. Extended producer responsibility is a product-
focused strategy that encourages environmentally friendly design and disposal of products through
transfer of this responsibility to product producers.
• The intergovernmental Organisation for Economic Co-operation and Development’s (OECD) definition
of extended producer responsibility identifies two specific features: the shifting of responsibility for
disposal “upstream” from municipalities to producers and encouragement through incentives to make the
design of products more environmentally friendly.

5 www.visionias.in ©Vision IAS

All Exam Study Material Available On - PHOTOCOPYNOTES.COM


Join Our Telegram Group - t.me/pdf4me
Q 15.A
• Carbon monoxide (CO) is one of the most serious air pollutants. It is a colourless and odourless gas,
highly poisonous to living beings because of its ability to block the delivery of oxygen to the organs and
tissues. It is produced as a result of incomplete combustion of carbon.
• Carbon monoxide is mainly released into the air by automobile exhaust. Other sources, which produce
CO, involve incomplete combustion of coal, firewood, petrol, etc.
• It binds to haemoglobin to form carboxyhaemoglobin, which is about 300 times more stable than
the oxygen-haemoglobin complex. In blood, when the concentration of carboxyhaemoglobin reaches
about 3–4 per cent, the oxygen carrying capacity of blood is greatly reduced. This oxygen deficiency,
results into headache, weak eyesight, nervousness and cardiovascular disorder. This is the reason why
people are advised not to smoke. In pregnant women who have the habit of smoking the increased CO
level in blood may induce premature birth, spontaneous abortions and deformed babies. Hence option (a)
is the correct answer.
• Sulfur dioxide is a nonflammable, colorless gas with a very strong, pungent odour.
• Carbon dioxide is colorless and odourless but it is produced by the complete combustion of carbon.
• Ammonia is a colourless gas with a characteristic pungent smell

Q 16.A
• Cenospecies: They are separate species of organisms that are related through their capability of
interbreeding, such as dogs and wolves. A cenospecies contains all those ecospecies so related that they
are able to exchange genes among themselves to a limited extent through hybridization. Hence option (a)
is the correct answer.
• Cryptic species: The species which are alike on the basis of observed features but are genetically and
sexually they are different are cryptic species. There is a confusion between the terms sibling species and
cryptic species. The cryptic species are incapable of interbreeding but the sibling species can interbreed
and are incapable of producing fertile hybrids.
• Monotypic species: When a genus includes a single species but does not include any subspecies, e.g.,
Vampyroteuthis, a vampire squid which is a single monotypic genus and also contains a single species, V.
infernalis (monotypic species). Blackwelder (1967) states that the species with a single subspecies, called
monotypic species.
• Sibling species: Two or more than two closely related species which are morphologically alike but
behaviourally or reproductively isolated from each other. Examples are Drosophila persimilis and D.
pseudoobscura. The mosquito Anopheles maculipennis complex consists of several subspecies, of which a
few are vector of malaria and the rest are harmless.

Q 17.C
• FSSAI is creating an ecosystem of Food Safety Mitras (FSMs) who will help food business operators
(FBOs) with licensing and registration, training and auditing hygiene at different institutions such as
schools, colleges and corporate campuses.
• The FSM scheme will help to strengthen food safety administration and scale up the ‘Eat Right India’
movement.
• The ‘Food Safety Mitra (FSM)’ scheme will support small and medium scale food businesses to
comply with the food safety laws and facilitate licensing and registration, hygiene ratings and training.
Apart from strengthening food safety, this scheme would also create new employment opportunities for
youth, particularly with food and nutrition background. The FSMs would undergo training and
certification by FSSAI to do their work and get paid by food businesses for their services.
• The scheme has no provision related to testing of GM crops.
• Hence only statements 1 and 2 are correct.

Q 18.B
• Thiruvalluvar is a celebrated ancient Tamil poet and philosopher, and the author of the Thirukkural, a
collection of rhyming couplets. His work has been acclaimed by philosophers, theologians and literary
savants around the world. Saint Thiruvalluvar is believed to have been born near Chennai in BC 30,
though exact time remains disputed.
• Tyagaraja was a renowned composer of Carnatic music, born in 1767. Hence, statement 1 is not correct.
• Thiruvalluvar composed Thirukkural, an ancient treatise on the Code of Ethics and Universal
Human Values, in Tamil. Opinions are divergent regarding the age of Thirukkural, yet, majority concurs
to that of, between 1st century BC and 2nd AD. This is adduced by the mention of Kural verses in

6 www.visionias.in ©Vision IAS

All Exam Study Material Available On - PHOTOCOPYNOTES.COM


Join Our Telegram Group - t.me/pdf4me
Silapathikaram and Manimekalai, which were written during Tamil Sangam period of 2nd Century
AD. Hence, statement 2 is correct.
• Recently, Thiruvalluvar was in the news for the tug of war going on with regard to his religious or
sectarian affiliations. Some scholars consider him to be a proponent of Jainism and other consider
him to be a propounder of Christianity.

Q 19.D
• A microbial fuel cell (MFC) is a bio-electrochemical device that harnesses the power of respiring
microbes to convert organic substrates directly into electrical energy. At its core, the MFC is a fuel
cell, which transforms chemical energy into electricity using oxidation-reduction reactions. Microbial
fuel cells are devices that use microbes, such as bacteria, as the catalysts to oxidize organic and inorganic
matter and generate current.
• Microbial fuel cells rely on living biocatalysts to facilitate the movement of electrons throughout their
systems instead of the traditional chemically catalyzed oxidation of fuel at the anode and reduction at the
cathode. The most promising MFC's for commercialization in today's energy industry are mediators
MFC's which use a special type of microorganism termed exoelectrogens. Exoelectrogens are
electrochemically active bacteria.
• The operational and functional advantages of MFCs are:
o MFCs use organic waste matter as fuels and readily available microbes as catalysts.
o MFCs do not require highly regulated distribution systems like the ones needed for Hydrogen Fuel
Cells.
o MFCs have high conversion efficiency as compared to Enzymatic Fuel Cells, in harvesting up to 90%
of the electrons from the bacterial electron transport system.
• Microbial fuel cells can be used in a variety of applications like:
o to power a wide range of vital conservation tools remotely, including sensors, monitoring platforms,
and camera traps.
o in wastewater treatment: conditions of a wastewater treatment plant are ideal for the types of bacteria
that can be used in an MFC. Exoelectrogens are more than happy to breakdown and metabolize the
carbon-rich sewage of a wastewater stream to produce electrons that can stream into a cheap
conductive carbon cloth anode.
o Powering underwater monitoring devices.
o Power supply to remote sensors.
o BOD sensing- to use it as a sensor for pollutant analysis and in situ process monitoring and control.
o Bio-hydrogen production
o for the remediation of various environmental pollutants viz. antibiotics, synthetic dyes, phenolic
compounds, nitrogen-based compounds, ethyl acetate, toluene, polycyclic aromatic hydrocarbons,
perchlorate, pesticide, sulphur, emerging contaminants, trace organic compounds etc.

Q 20.D
• Permafrost is perennially frozen soil that has been below 0oC for at least two years. Permafrost is a
condition where top layer upto depth of 20-40 cm is completely frozen. This happens in high latitude
and altitude regions.
• It's found underneath about 25% of the northern hemisphere, mainly around the Arctic - but also in the
Antarctic and Alpine regions. In the northern region of Alaska, the permafrost has been warming at about
one-tenth of a degree Celsius per year since the mid 2000s.
• These top soils melt due to global warming which is a great threat as there melt will aggravate the
condition of global warming. As permafrost thaws, this carbon is released to the atmosphere in the form
of methane, a powerful greenhouse gas. This process leads to more climate change and is an example of a
positive feedback loop, which happens when warming causes changes that lead to even more warming.
• As permafrost thaws, it can also cause substantial changes in the local ecosystem, altering the flow
of water atop and through the soil, as well as what plant and animal life can thrive in the area.
• It is a part of life in the frigid North Slope, underlying buildings, roads and other infrastructure. When it
thaws, it can cause considerable damage. In Alaska, the warming of the permafrost has been linked to
trees toppling, roads buckling and the development of sinkholes.
• When permafrost melts, the land above it sinks or changes shape. Sinking land can damage buildings
and infrastructure such as roads, airports, and water and sewer pipes. It also affects ecosystems.

7 www.visionias.in ©Vision IAS

All Exam Study Material Available On - PHOTOCOPYNOTES.COM


Join Our Telegram Group - t.me/pdf4me
Q 21.B
• The Union Minister of Earth Sciences has recently launched the Gagan Enabled Mariner’s Instrument for
Navigation and Information (GEMINI) device.
• GEMINI is a new satellite-based communication system that will alert deep-sea fishermen of upcoming
disasters. It is a low cost device having a box-shaped receiver that has an antenna and in-built battery that
can last three to four days.
• It is based on GAGAN (GPS-Aided Geo Augmented Navigation) satellite system developed by the Indian
Space Research Organization (ISRO) and AAI. The GAGAN satellite system consists of three
geosynchronous satellites namely the GSAT-8, GSAT-10 and GSAT-15 and these three cover the entire
Indian Ocean region constantly.
• Statement 1 is not correct: GEMINI device only allows one-way communication, that a fisherman only
received the warning related to cyclones, high waves and tsunamis in the sea. It can’t be used by
fishermen to make calls. The GEMINI device receives and transfers the data received from GAGAN
satellite/s to a mobile through Bluetooth communication. A mobile application developed by INCOIS
decodes and displays the information in nine regional languages.
• Statement 2 is correct: It is developed by the Indian National Centre for Ocean Information Services
(INCOIS) in collaboration with the Airports Authority of India (AAI).
• Presently, the fishermen receive advisories, forecasts and early warnings from a wide range of
mechanisms such as Potential Fishing Zones (PFZ) advisories, ocean state forecasts, high wave alerts,
tsunami and storm surge early warning services. However, all these messages can only get transmitted up
to 10 to 12 km from the coast. GEMINI device will help in disseminating the information to
fishermen who go in the sea beyond 50 nautical miles; sometimes up to 300 nautical miles and
beyond to conduct multi day fishing.

Q 22.D
• Bioindicators are organisms, such as lichens, birds and bacteria, that are used to monitor the health of the
environment. The organisms and organism associations are monitored for changes that may indicate a
problem within their ecosystem. The changes can be chemical, physiological or behavioural.
Bioindicators are relevant for Ecological health.
• Bioindicators can be a measure, an index of measures, or a model that characterizes an ecosystem or one
of its critical components. They are also a method of monitoring or detecting the negative impacts that
industrial activity has on the environment. This information helps develop strategies that will prevent or
lower such effects and make the industry more sustainable. The role of bioindicators in sustainable
development will help ensure that industry leaves the smallest footprint possible on the environment.
Role of Bioindicators
• If toxins are present, certain plants may not be able to grow in the area affected.
• Monitoring population numbers of animals may indicate damage to the ecosystem in which they live.
• Algae blooms are often used to indicate large increases of nitrates and phosphates in lakes and rivers.
• If pollution causes the reduction of an important food source, the animals dependent on it for food
may also decrease. Animals may also change their behaviour or physiology if a toxin is present.
• The levels of certain liver enzymes in fish increase if they are exposed to pollutants in the water.
• Few tolerant species like tubifex and insect larvae may survive in highly polluted water with low
dissolved oxygen content. Hence they indicate polluted water
• Microorganisms can also be used as indicators of toxins in an ecosystem. Some microorganisms will
produce stress proteins if exposed to certain pollutants. By measuring the levels of stress proteins, we
can get an idea of the level of pollution present in the environment.
Examples of bioindicators : lichens , tubifex , insect larvae , frog , algal blooms etc.
Hence all the statements are correct.

Q 23.D
• The Scheduled Castes and the Scheduled Tribes (Prevention of Atrocities) Amendment Bill, 2018
was introduced in Lok Sabha by the Minister for Social Justice and Empowerment August 3,
2018 which sought to amend the Scheduled Castes and the Scheduled Tribes (Prevention of
Atrocities) Act, 1989.
o The Act prohibited the commission of offences against members of the Scheduled Castes and
Scheduled Tribes and established special courts for the trial of such offences and the rehabilitation of
victims.
8 www.visionias.in ©Vision IAS

All Exam Study Material Available On - PHOTOCOPYNOTES.COM


Join Our Telegram Group - t.me/pdf4me
o In 2018, the Supreme Court stated that for persons accused of committing an offence under the
Act, approval of the Senior Superintendent of Police will be required before an arrest is made.
o Further, the Deputy Superintendent of Police may conduct a preliminary enquiry to find out
whether there is a prima facie case under the Act. The Bill stated that the investigating officer will
not require the approval of any authority for the arrest of an accused.
o Further, it provides that a preliminary enquiry will not be required for the registration of a First
Information Report against a person accused under the Act.
o The Act stated that persons accused of committing an offence under the Act cannot apply for
anticipatory bail. The Bill sought to clarify that this provision will apply despite any judgements or
orders of a court that provide otherwise.
• However, the Scheduled Castes and the Scheduled Tribes (Prevention of Atrocities) Amendment
Act, 2018 was enacted to reverse these changes. Section 18A was inserted in the Scheduled Castes and
the Scheduled Tribes (Prevention of Atrocities) Act, 1989, which provided that:
o preliminary enquiry shall not be required for registration of a First Information Report against
any person; (hence statement (d) is the correct answer) or
o the investigating officer shall not require approval for the arrest, if necessary, of any person, against
whom an accusation of having committed an offence under this Act has been made and no procedure
other than that provided under this Act or the Code shall apply.
• Recently, the Supreme Court recalled its 2018 order that diluted provisions of the Scheduled Castes and
Scheduled Tribes (Prevention of Atrocities) Act, 1989.

Q 24.A
• Sulfur dioxide is a gas released by both human and natural sources. It is a colourless gas with a
pungent, irritating odour and taste. Sulfur dioxide is used in many industrial processes such as chemical
preparation, refining, pulp-making and solvent extraction. In addition, it is used in the preparation and
preservation of food due to its ability to prevent bacterial growth and browning of fruit.
• Human Sources
o Burning of fossil fuels such as coal, oil and natural gas are the main source of sulfur dioxide
emissions. Coal fired power stations, in particular, are major sources of sulfur dioxide, with coal-
burning accounting for 50 per cent of annual emissions, as explained by the Tropospheric Emission
Monitoring Internet Service (TEMIS). Moreover, oil-burning accounts for a further 25-30 per cent.
o Sulfur dioxide emissions are released primarily as a result of generated electricity through fossil fuel
burning power stations. Additional smaller sources of sulfur dioxide are released from industrial
processes. These include extracting metal from ore and the burning of fuels with a high sulfur content
by locomotives, large ships and non-road equipment.
• Natural Sources
o Volcanic eruptions release large quantities of sulfur dioxide into the air. The vast quantities of sulfur
dioxide released during one eruption can be enough to alter the global climate. Similarly, hot springs
release sulfur dioxide into the atmosphere. Sulfur dioxide can even be produced by the reaction of
hydrogen sulfide with the oxygen in the air. Hydrogen sulfide is released from marshes and regions
in which biological decay is taking place.
Q 25.A
• Eutrophication is the natural aging of a lake by biological enrichment of its water.
• With time, streams draining into the lake introduce nutrients such as nitrogen and phosphorus, which
encourage the growth of aquatic organisms. As the lake’s fertility increases, plant and animal life
burgeons, and organic remains begin to be deposited on the lake bottom. Over the centuries, as silt and
organic debris pile up, the lake grows shallower and warmer, with warm-water organisms supplanting
those that thrive in a cold environment. Marsh plants take root in the shallows and begin to fill in the
original lake basin. Eventually, the lake gives way to large masses of floating plants (bog), finally
converting into land. Hence, statement 1 and 2 are correct.
• Chlorofluorocarbons do not play any role in Eutrophication. Hence, statement 3 is not correct.

Q 26.C
• The process of identification and enumeration of plant and animal species of an ecosystem gives its
species composition.
• The vertical distribution of different species occupying different levels is called stratification. For
example, trees occupy top vertical strata or layer of a forest, shrubs the second and herbs and grasses
occupy the bottom layers.
• Hence option (c) is the correct answer.
9 www.visionias.in ©Vision IAS

All Exam Study Material Available On - PHOTOCOPYNOTES.COM


Join Our Telegram Group - t.me/pdf4me
Q 27.B
• Recent context- Delhi municipal corporation has started the process of “biomining and bioremediation”
of the three landfills in Delhi namely Bhalswa, Okhla and Ghazipur.
• Biomining is the process of using microorganisms (microbes) to extract metals of economic interest
from rock ores or mine waste. Biomining techniques may also be used to clean up sites that have
been polluted with metals. Hence statement (b) is the correct answer.
• Valuable metals are commonly bound up in solid minerals. Some microbes can oxidize those metals,
allowing them to dissolve in water. This is the basic process behind most biomining, which is used for
metals that can be more easily recovered when dissolved than from the solid rocks.
• A different biomining technique, for metals which are not dissolved by the microbes, uses microbes to
break down the surrounding minerals, making it easier to recover the metal of interest directly from the
remaining rock.
• Most current biomining operations target valuable metals like copper, uranium, nickel, and gold that are
commonly found in sulfidic (sulfur-bearing) minerals.
• Microbes are especially good at oxidizing sulfidic minerals, converting metals like iron and copper into
forms that can dissolve more easily.
• Other metals, like gold, are not directly dissolved by this microbial process, but are made more accessible
to traditional mining techniques because the minerals surrounding these metals are dissolved and removed
by microbial processes.
• When the metal of interest is directly dissolved, the biomining process is called “bioleaching,” and when
the metal of interest is made more accessible or “enriched” in the material left behind, it is
called “biooxidation.” Both processes involve microbial reactions that can happen anywhere the
microbes, rocks, and necessary nutrients, like oxygen, occur together.
• Bioremediation is the treatment of pollutants or waste (as in an oil spill, contaminated groundwater, or an
industrial process) by the use of microorganisms (such as bacteria) that break down the undesirable
substances.

Q 28.D
• The UN Intergovernmental Panel on Climate Change's Special Report on the Ocean and Cryosphere in
a Changing Climate assesses the impacts of climate change on ocean, coastal, polar and mountain
ecosystems, and the human communities that depend on them.
• According to the report, the global ocean has warmed unabated since 1970 and has taken up more than
90% of the excess heat in the climate system, with consequences now visible in increased ocean
acidification, stratification and loss of oxygen.
• The report underscores the urgency of prioritizing “timely, ambitious and coordinated action” to address
“unprecedented” and enduring changes in the ocean and cryosphere.
• According to the report, global warming has already reached 1°C above preindustrial levels, with:
profound consequences for ecosystems and people; a warmer, more acidic and less productive ocean;
melting glaciers and ice sheets causing increased sea level rise; and coastal extreme events becoming
more severe.

Q 29.B
• Warm-blooded creatures, like mammals and birds, try to keep the inside of their bodies at a constant
temperature. They do this by generating their own heat when they are in a cooler environment, and by
cooling themselves when they are in a hotter environment. To generate heat, warm-blooded animals
convert the food that they eat into energy. They have to eat a lot of food, compared with cold-blooded
animals, to maintain a constant body temperature. Hence statement 3 is correct.
• Cold-blooded creatures take on the temperature of their surroundings. They are hot when their
environment is hot and cold when their environment is cold. Thus they match the temperature of their
environment. Hence statement 1 is correct.
• Humans are warm-blooded endotherms like other mammals and birds. Cold-blooded animals such as
amphibians and reptiles must constantly move in and out of sunlight, trying to maintain their body
temperature. Hence statement 2 is not correct.

Q 30.C
• Usually, species having a wide range of distribution evolve genetically adapted local populations, called
ecotypes. Ecotypes differ from each other on the basis of morphological and physiological characters.
Hence, statement 1 is correct.

10 www.visionias.in ©Vision IAS

All Exam Study Material Available On - PHOTOCOPYNOTES.COM


Join Our Telegram Group - t.me/pdf4me
• Ecotypes are inter-fertile. Though the different ecotypes of a species are morphologically and genetically
distinct, yet because of their inter-fertility, they are put into one taxonomic species. Hence, statement 2 is
correct.

Q 31.B
• Ozone is a gas made up of three oxygen atoms (O3). It occurs naturally in small (trace) amounts in the
upper atmosphere (the stratosphere). Ozone protects life on Earth from the Sun’s ultraviolet (UV)
radiation. Although ozone in the upper atmosphere provides protection from UV rays, ozone at ground-
level is a toxic substance that is harmful to humans to breathe.
• Ozone can be formed when volatile organic compounds (including those from plants, automobile,
and industrial sources) combine with nitrogen oxide emissions (often from vehicles or fuel
combustion) in the presence of sunlight.
• Urban trees reduce air pollution by having particulates and gases deposit on their leaves and branches. On
the other hand, trees can potentially reduce air quality through a range of mechanisms, one being the
emission of organic compounds that can lead to ozone formation. The Volatile organic compounds
(VOC) emitted by urban plants could trigger the formation of ground-level (bad) ozone. A plant's
ozone-forming potential is determined by its ability to release the VOCs into the atmosphere.
• Hence option (b) is the correct answer.

Q 32.A
• In 2009, a group of 28 internationally renowned scientists proposed to identify the nine processes that
regulate the stability and resilience of the Earth system.
• The scientists proposed quantitative planetary boundaries within which humanity can continue to develop
and thrive for generations to come. Crossing these boundaries increases the risk of generating large-
scale abrupt or irreversible environmental changes. Since then the planetary boundaries framework
has generated enormous interest within science, policy, and practice.
• According to the paradigm, transgressing one or more planetary boundaries may be deleterious or even
catastrophic due to the risk of crossing thresholds that will trigger non-linear, abrupt environmental
change within continental- to planetary-scale systems. The Earth system process boundaries mark the
safe zone for the planet to the extent that they are not crossed.

Q 33.A
• Ornamental fishery is a sub-sector of the fisheries sector dealing with breeding and rearing of coloured
fish of both freshwater and marine water. Ornamental fishes are usually kept in glass aquarium and hence
popularly known as 'Aquarium Fishes'. They need not always have bright colours; as sometimes their
peculiar characteristics such as body colour, morphology, mode of taking food etc. may also add to their
attractiveness. Though ornamental fisheries does not directly contribute to the food and nutritional
security, it generates livelihood and income for the rural and peri-urban population, especially women and
unemployed youth as part-time activities. The ornamental fish industry in India is small but vibrant, with
potential for tremendous growth. About 400 species of marine ornamental fishes and 375 freshwater
ornamental varieties are available in various parts of our country.

Q 34.A
• Blue green algae: Pre-cambrian era
• Fish: Palaezoic era
• Frog: Mesozoic era
• Ape: Cainozoic era
• How to arrive at the answer:
• Bacteria and other unicellular organisms were the first species. Hence, options c and d can be eliminated.
Also, we are closest to ape genetically. Hence, it must be the last specie before humans.

Q 35.A
• The Sun provides the energy that drives Earth’s climate, but not all of the energy that reaches the top of
the atmosphere finds its way to the surface. That’s because aerosols—and clouds seeded by them—reflect
about a quarter of the Sun’s energy back to space.
• Although most aerosols reflect sunlight, some also absorb it. An aerosol’s effect on light depends
primarily on the composition and color of the particles. Broadly speaking, bright-colored or translucent
particles tend to reflect radiation in all directions and back towards space. Darker aerosols can absorb
significant amounts of light.
11 www.visionias.in ©Vision IAS

All Exam Study Material Available On - PHOTOCOPYNOTES.COM


Join Our Telegram Group - t.me/pdf4me
• Pure sulfates and nitrates reflect nearly all radiation they encounter, cooling the atmosphere.
• Salt particles tend to reflect all the sunlight they encounter, thus helps in cooling the earth.
• Aerosols can have a major impact on climate when they scatter light. In 1991, the eruption of Mount
Pinatubo in the Philippines ejected more than 20 million tons of sulfur dioxide—a gas that reacts with
other substances to produce sulfate aerosol—as high as 60 kilometers (37 miles) above the surface,
creating particles in the stratosphere. Those bright particles remained above the clouds and didn’t get
washed from the sky by rain; they settled only after several years.
• Climatologists predicted global temperatures would drop as a result of that global sulfate infusion.
• On the other hand all the green house gases (water vapour, carbon dioxide, methane, nitrous oxide,
fluorinated gases), warms the earth's atmosphere.

Q 36.C
• A herbaceous plant is a plant that does not have a persistent woody stem above ground and its stems are
green and soft. These plants grow fast and produce flowers and many seeds in a short period of time. Aloe
vera and Tulsi are herbaceous plants.
• Most herbaceous plants are small and their stems are not thick but there are some cases of big herbaceous
plants. Banana is a herbaceous plants but it appears like a tree.
• Neem is a tree in the mahogany family. It has a woody stem.

Q 37.A
• Geothermal energy is generated in over 20 countries. The United States is the world’s largest producer,
and the largest geothermal development in the world is The Geysers north of San Francisco in
California. In Iceland, many of the buildings and even swimming pools are heated with geothermal hot
water. Iceland has at least 25 active volcanoes and many hot springs and geysers.
• There are many advantages of geothermal energy. It can be extracted without burning a fossil fuel such as
coal, gas, or oil. Geothermal fields produce only about one-sixth of the carbon dioxide that a relatively
clean natural-gas-fueled power plant produces. Binary plants release essentially no emissions. Unlike
solar and wind energy, geothermal energy is always available, 365 days a year. It’s also relatively
inexpensive; savings from direct use can be as much as 80 percent over fossil fuels.
• But it has some environmental problems. The main concern is the release of hydrogen sulfide, a gas
that smells like rotten egg at low concentrations. Another concern is the disposal of some
geothermal fluids, which may contain low levels of toxic materials. Although geothermal sites are
capable of providing heat for many decades, eventually specific locations may cool down.
• Hence only statement 1 is correct.

Q 38.B
• Ministry of Finance launched Sabka Vishwas-Legacy Dispute Resolution Scheme, 2019. The objective of
the Scheme is to free a large segment of the taxpayers from the legacy taxes. The Scheme is specially
tailored to free a large number of small taxpayers of their pending disputes with the tax administration.
• The scheme will help the taxpayers for closing their pending disputes relating to legacy Service Tax
and Central Excise cases that are now subsumed under GST so they can focus on GST.
• Dispute resolution and amnesty are the two main components of the Scheme:1. The dispute resolution
component is aimed at liquidating the legacy cases of Central Excise and Service Tax that are subsumed
in GST and are pending in litigation at various forums.2. The amnesty component of the Scheme offers an
opportunity to taxpayers to pay the outstanding tax and be free of any other consequences under the law.
• The most attractive aspect of the Scheme is that it provides substantial relief in the tax dues for all
categories of cases as well as full waiver of interest, fine, penalty. In all these cases, there would be no
other liability of interest, fine or penalty. There is also a complete amnesty from prosecution.

Q 39.A
• Carbon in the Earth's atmosphere exists in two main forms: carbon dioxide and methane. Both of
these gases absorb and retain heat in the atmosphere and are partially responsible for the greenhouse
effect. Methane produces a larger greenhouse effect per volume as compared to carbon dioxide, but it
exists in much lower concentrations and is more short-lived than carbon dioxide, making carbon dioxide
the more important greenhouse gas of the two. Increase in average global temperature causes glacial
melt. Hence statement 2 is correct.
• Ocean acidification is a direct consequence of increased human induced carbon dioxide concentrations in
the atmosphere. The ocean absorbs over 25% of all anthropogenic emissions from the atmosphere each
year. As CO2 dissolves in sea water it forms carbonic acid, thereby decreasing the ocean’s
12 www.visionias.in ©Vision IAS

All Exam Study Material Available On - PHOTOCOPYNOTES.COM


Join Our Telegram Group - t.me/pdf4me
pH, leading to a suite of changes collectively known as ocean acidification. Hence statement 3 is not
correct.
• Carbon dioxide is essential for plant and phytoplankton growth. An increase in carbon dioxide
could increase growth by fertilizing those few species of phytoplankton and ocean plants (like sea
grasses) that take carbon dioxide directly from the water. With more atmospheric carbon dioxide
available to convert to plant matter in photosynthesis, plants will be able to grow more. This
increased growth is referred to as carbon fertilization. Hence statement 1 is not correct.

Q 40.D
• Climate feedbacks: These are the processes that can either amplify or diminish the effects of climate
forcings. Feedback that increases an initial warming is called "positive feedback." A feedback that
reduces an initial warming is a "negative feedback." Hence statement 1 is not correct.
• Scientists are aware of a number of positive feedbacks loops in the climate system. One example is
melting of ice. The ice albedo effect is simply a name for how ice and snow reflect solar radiation,
and thus help keep the Earth cool. Since a cool Earth also tends to have more ice and snow, the ice
albedo effect is an example of a positive climate feedback. Because ice is light-coloured and reflective,
a large proportion of the sunlight that hits it is bounced back to space, which limits the amount of
warming it causes. But as the world gets hotter, ice melts, revealing the darker-coloured land or water
below. The result is that more of the sun's energy is absorbed, leading to more warming, which in turn
leads to more ice melting and so on. Other examples of positive feedback mechanism include Water
vapour release, carbon release into the atmosphere etc. Hence statement 2 is not correct.
• However in case of cloud feedback, feedback type changes with behavior of clouds. Seen from below,
clouds emit infrared radiation back to the surface, and so exert a warming effect; seen from above, clouds
reflect sunlight and emit infrared radiation to space, and so exert a cooling effect. Whether the net effect
is warming or cooling depends on details such as the type and altitude of the cloud. High clouds tend
to trap more heat and therefore have a positive feedback, low clouds normally reflect more sunlight so
they have a negative feedback.

Q 41.C
• Lindeman in 1942 pointed out a series of generalizations about the relationships between trophic levels
within normal ecosystems. The main ones are:
o the more remote an organism is from its initial source of energy the less probable that it will be
dependent only on the preceding trophic level as a source of energy (that is, species) and above tend
to be generalists rather than specialists in terms of feeding habit.
o the relative loss of energy due to respiration is progressively greater for higher trophic levels.
o species at higher trophic levels appear to be progressively more efficient in using their available
food supply, because increased activity by predators increases their chances of encountering
suitable prey species, and in general predators are less specific than their prey in food preferences.
o Higher trophic levels tend to be less discrete than lower ones
o Food chains tend to be reasonably short. Four vertical links is a common maximum.
• Hence both the statements are correct.

Q 42.B
• The world polio day on October 24 marked an important milestone in the war against polio when the
Global Commission for the Certification of Poliomyelitis Eradication officially declared that wild
poliovirus type 3 has been eradicated. The last case of wild poliovirus type 3 was seen in northern
Nigeria in 2012.
• This is the second wild poliovirus to be declared eliminated — the first was in 2015 when type 2 wild
poliovirus was declared as eliminated.
• With two of the three wild polioviruses eliminated, only type 1 wild poliovirus is still in circulation and
is restricted to just two countries — Afghanistan and Pakistan. As on October 23, there were 18 cases
of polio caused by wild virus type 1 in Afghanistan and 76 polio cases in Pakistan this year. While the
number of cases reported this year from Afghanistan is quite close to the 21 reported last year, there has
been over six-fold increase in the number of cases in Pakistan.
• Though India has excellent polio immunisation coverage and measures have been put in place to prevent
the spread from polio-endemic countries, there is no room for complacency.
• Hence only statements 1 and 3 are correct.

13 www.visionias.in ©Vision IAS

All Exam Study Material Available On - PHOTOCOPYNOTES.COM


Join Our Telegram Group - t.me/pdf4me
Q 43.B
• Hydrogen is an environmentally friendly alternative to fossil fuels, and they can be used to power just
about any machine needing energy. The fuel cell, which is the energy conversion device that can capture
and use the power of hydrogen effectively is the key to making this happen.
• Hydrogen-powered fuel cells have two or three times the efficiency of traditional combustion
technologies. For example, a conventional combustion-based power plant usually generates electricity
between 33 to 35 per cent efficiency. Hydrogen fuel cells are capable of generating electricity of up to 65
per cent efficiency.
• Also, a gasoline-powered engine in a conventional car is not as efficient at converting chemical energy
into gasoline into power that moves vehicles under normal driving conditions. With vehicles that use
hydrogen fuel cells, and also use electric motors, are more efficient as they can use 40 to 60 per cent of
the fuel’s energy. As a result, there is more than a 50% reduction in fuel consumption.
• Advantages of Hydrogen Fuel Cells:
o It is readily available: As mentioned earlier, hydrogen is a basic earth element and it’s very
abundant. However, it takes a whole lot of time to separate hydrogen gas from its companion
substances. While that may be the case, the results produce a powerful clean energy source.
o It doesn’t produce harmful emissions: When hydrogen is burned, it doesn’t emit harmful
substances. Basically, it reacts to oxygen without burning and the energy it releases can be used to
generate electricity used to drive an electric motor. Also, it doesn’t generate carbon dioxide when
burnt, not unlike other power sources.
o Environmentally friendly: Hydrogen is a non-toxic substance which is rare for a fuel source. Others
such as nuclear energy, coal and gasoline are either toxic or found in places that have hazardous
environments. Because hydrogen is friendly towards the environment, it can be used in ways that
other fuels can’t even possibly match.
o It can be used as fuel in rockets: Hydrogen is both powerful and efficient. It is enough to provide
power for powerful machines such as spaceships. Also, given that it is environmentally friendly, it is a
much safer choice compared to other fuel sources. A fun fact: hydrogen is three times as powerful as
gasoline and other fossil fuels. This means that it can accomplish more with less.
o It is fuel-efficient: Compared to diesel or gas, hydrogen is much more fuel-efficient as it can produce
more energy per pound of fuel. This means that if a car is fueled by hydrogen, it can go farther than a
vehicle loaded with the same amount of fuel but using a more traditional source of energy.
o It is renewable: Hydrogen can be produced again and again, unlike other non-renewable sources of
energy. This means that with hydrogen, you get a fuel source that is limited. Basically, hydrogen
energy can be produced on demand. Also, it is widely available – all that is needed is to break the
water molecules so it gets separated from oxygen. It’s without question a time-consuming process but
the outcome is great.
• Disadvantages of Hydrogen Fuel Cells
o Expensive: While widely available, hydrogen is expensive. A good reason for this is that it takes a lot
of time to separate the element from others. If the process were really simple, then a lot would have
been doing it with relative ease, but it’s not.
o Difficult to store: Hydrogen is very hard to move around. When speaking about oil, that element can
be sent though pipelines. When discussing coal, that can be easily carried off on the back of trucks.
When talking about hydrogen, just moving even small amounts is a very expensive matter. For that
reason alone, the transport and storage of such a substance is deemed impractical.
o Highly flammable: Since it is a very powerful source of fuel, hydrogen can be very flammable. In
fact, it is on the news frequently for its many number of risks. Hydrogen gas burns in air at very wide
concentrations – between 4 and 75 percent.
o Dependent on fossil fuels: Although hydrogen energy is renewable and has minimal environmental
impact, other non-renewable sources such as coal, oil and natural gas are needed to separate it from
oxygen. While the point of switching to hydrogen is to get rid of using fossil fuels, they are still
needed to produce hydrogen fuel.

Q 44.A
• Acid rain, or acid deposition, is a broad term that includes any form of precipitation that contains acidic
components, such as sulfuric acid or nitric acid, according to the Environmental Protection Agency
(EPA).
• Acid rain is caused by a chemical reaction that begins when compounds like sulfur dioxide and nitrogen
oxides are released into the air. These substances can rise very high into the atmosphere, where they mix
and react with water, oxygen, and other chemicals to form more acidic pollutants, known as acid rain.
14 www.visionias.in ©Vision IAS

All Exam Study Material Available On - PHOTOCOPYNOTES.COM


Join Our Telegram Group - t.me/pdf4me
• Sulfur dioxide and nitrogen oxides dissolve very easily in water and can be carried very far by the wind.
As a result, the two compounds can travel long distances where they become part of the rain, sleet, snow,
and fog that we experience on certain days.
Sources of compounds causing acid rain:
(A) Sulphur
Natural Resources
• Seas and oceans
• Volcanic eruptions
• Biological process in the soil e.g., Decomposition of Organic matter.
Man- made sources:
• burning of coal (60%) and
• Petroleum products (30 % of S02) and smelting of metal sulphides ores to obtain the pure metals
• Industrial production of sulphuric acid in metallurgical, chemical and fertilizers industries.
(B) Nitrogen
Natural resource
Lightning
• Volcanic eruption
• Biological activities
Anthropogenic sources:
• Forest fires
• Combustion of Oil, Coal, And Gas
(C) Formic acid
• Biomass burning due to forest fires cause emission of formic acid (HCOOH) and formaldehyde
(HCHO) into the atmosphere.
• Large fraction formaldehyde gets photo-oxidation and forms formic acid in the atmosphere.
• These are the main compounds that cause acidification of rain in the atmosphere.
(D) Other Acids:
• Chlorine
• Phosphoric Acid
• Hydrochloric acid (smokestacks)
• Carbon monoxide and carbon dioxide. These become carbonic acids.

Q 45.A
Soil Steaming is a method of soil sterilization. It is the act of killing or destroying the action of diseases
causing organism that are present in the soil without causing harm to other plants.

Q 46.B
• In ecology, a mesic habitat is a type of habitat with a moderate or well-balanced supply of
moisture, e.g., a mesic forest, a temperate hardwood forest, or dry-mesic prairie. Mesic habitats transition
to xeric shrublands in a non-linear fashion, which is evidence of a threshold. Further examples of mesic
habitats include streamsides, wet meadows, springs, seeps, irrigated fields, and high elevation habitats.
These habitats effectively provide drought insurance as land at higher elevations warms due to seasonal or
other change.
• Healthy mesic habitats act like sponges in that they store water in such a way that it can be deposited to
neighbouring habitats as needed. They are common in dryer regions of the western United States and can
be a good water source to neighbouring desert habitats. Healthy mesic habitats also provide forb and
insects for organisms belonging to higher trophic levels, such as grouse.
• A hot desert is deficient in water supply. Coastal Mangroves and Estuaries have plenty supply of water
and are hydric in nature.

Q 47.D
The characteristics of a species particularly susceptible to extinction are:
• Large body size (Bengal tiger, Lion and elephant)
• Small population size and low reproductive rate (Blue Whale and Giant Panda).
15 www.visionias.in ©Vision IAS

All Exam Study Material Available On - PHOTOCOPYNOTES.COM


Join Our Telegram Group - t.me/pdf4me
• Feeding at high trophic levels in the food chain (Bengal Tiger and Bald eagle).
• Fixed migratory routes and habitat (Blue Whale and Whooping Crane).
• Localized and narrow range of distribution (woodland caribou; many island species).

Q 48.A
• Pobitora Wildlife Sanctuary is situated in the flood plains of River Brahmaputra in the district of
Morigaon and about 45 kms from Guwahati. Pobitora originally was a grazing reserve for cattle of
erstwhile Nagaon district, came into limelight during the year 1961-62 for sighting of One Horned Rhino
(Rhinoceros unicornis). During that time a few rhinos strayed out of Lawkhowa and Orang Wild Life
Sanctuaries and gradually became resident. Today, it harbours the highest density of Rhino in the world
and second highest concentration of Rhino in Assam after Kaziranga National Park.
• Besides rhinoceros, the other mammals found are Leopard, Leopard cat, Fishing cat, Jungle cat,
Feral Buffalo, Wild pigs, Chinese pangolins etc.
• Pobitora Wildlife Sanctuary is also an 'Important Bird Area' declared by Birdlife International. In
winter, Pobitora becomes a birder’s heaven, with thousands of waterfowl thronging the wetlands.
• Recently, the carcasses of the feral buffaloes were found at the site. Veterinarians have confirmed
anthrax as the cause of death. Anthrax, caused by the bacterium Bacillus anthracis, can be fatal for
humans who come in contact with infected animals. It is characterised by blisters around swellings on the
skin, chest pain, vomiting, diarrhoea and fever.
Q 49.A
• Also known as bloom dilution, the process of biodilution is the decrease in concentration of a substance
(especially pollutants, like heavy metals) with an increase in trophic level in food chain. This phenomenon
is observed mostly in eutrophic (nutrient-rich and highly productive) water environments. For example,
during algal blooms an increase of algae reduces the concentration of pollutants a consequence of their
larger biomass refers to a decrement in the concentration or quantity of a pollutant when that
particular pollutant gains a higher trophic level through consumption. Studies of bioaccumulation
have led to researchers discovering the importance of biodilution. Hence option (a) is the correct
answer.
• A large percentage of studies conducted have almost invariably concluded that there is a relationship
between biodilution and bioaccumulation. Zooplankton that live in places that have higher amounts of
nutrients and productivity have been shown to contain lesser amounts of mercury compared to
those that do not.
• If creatures in the lower trophic levels multiply exponentially during a bloom, then the heavy metals
absorbed by them will, in turn, reduce since there are more absorbers. Consequently, when these primary
producers are eaten by those organisms occupying higher levels, they transfer fewer amounts of the
heavy metals. This process creates a ripple effect that goes all the way to the top consumers, such as
humans. The overall effect is known as biodilution. Some of the organisms that will experience a
reduction in pollutants after the algal bloom include the likes of daphnia and zooplankton.

Q 50.D
• Bees are disappearing at an alarming rate due to the excessive use of pesticides in crops, diseases, growth
of parasites that capture bee colonies. It's true that the extinction of bees would mean the end of
humanity. Out of the 100 crop species that provide us with 90% of our food, 35% are pollinated by bees,
birds and bats.
• Option 2 is correct: Some plants are pollinated by wind, but that rate is very slow. Insects are the
primary pollinators on the planet. Beetles and butterflies also pollinate, but bees are the most efficient
insects for this purpose. Some of the crops benefited by bee pollination include:
o Fruits and nuts: Almond, apple, apricot, peach, strawberry, citrus and litchi
o Vegetable and Vegetable seed crops: Cabbage, cauliflower, carrot, coriander, cucumber, melon,
onion, pumpkin, radish and turnip.
o Oilseed crops: Sunflower, niger, rapeseed, mustard, safflower, gingelly. If bees went extinct, there
would be a massive decline in the production of crops.
• Option 1 is correct: Herbivores, who depend on certain plant species, will be affected first. They would
go extinct if plants ceased to exist. For example, many cattle used for milk and meat depend on grasses
like alfalfa and lupins, both of which depend on insect pollination. Due to the declining population of
herbivores, tertiary carnivores will begin to suffer immediately. The only beneficiaries from this scenario
would be scavengers (eagles, vultures, ravens etc.)
• Option 3 is correct: Wild bees pollinate flowers, trees and shrubs, which in turn provide flood control,
prevent soil erosion and help regulate the climate. Seed and shelter other native wildlife.
16 www.visionias.in ©Vision IAS

All Exam Study Material Available On - PHOTOCOPYNOTES.COM


Join Our Telegram Group - t.me/pdf4me
Q 51.D
• Union Ministry of Minority Affairs launched the Hunar Haat initiative. It aims to provide
employment and employment opportunities to lakhs of artisans, craftsmen and traditional culinary
experts through “Hunar Haat” in the next 5 years.
• The next “Hunar Haat” will be organised at North Central Zone Cultural Centre in Prayagraj (UP) from
01st to 10th November 2019 where more than 300 master artisans and culinary experts including a large
number of women artisans from every corner of the country will participate. All the “Hunar Haat”, to be
organised in 2019 and 2020 will be based on the theme of “Ek Bharat Shresth Bharat”.
• The “Hunar Haat” has proved to be an effective programme to provide employment and employment
opportunities to master artisans. More than 2 lakh 50 thousand master artisans, craftsmen and culinary
experts have been provided employment and employment opportunities in the last 3 years. These include
a large number of women.

Q 52.C
• High-altitude adaptation in humans is an instance of evolutionary modification in certain human
populations, including those of Tibet in Asia, the Andes of the Americas, and Ethiopia in Africa,
who have acquired the ability to survive at extremely high altitudes.
• People living in high altitude regions have undergone extensive physiological and genetic changes,
particularly in the regulatory systems of oxygen respiration and blood circulation, when compared to the
general lowland population.
• Tibetans inhale more air with each breath and breathe more rapidly than either sea level populations or
Andeans. Tibetans have better oxygenation at birth, enlarged lung volumes throughout life, and a higher
capacity for exercise. They show a sustained increase in cerebral blood flow, and less susceptibility to
chronic mountain sickness than other populations, due to their longer history of high-altitude
habitation. Hence statement 3 is correct.
• In addition, Tibetans have a second biological adaptation, which expands their blood vessels, allowing
them to deliver oxygen throughout their bodies more effectively than sea-level people do. Tibetans'
lungs synthesize larger amounts of a gas called nitric oxide from the air they breathe. One effect of nitric
oxide is to increase the diameter of blood vessels, which suggests that Tibetans may offset low oxygen
content in their blood with increased blood flow. Hence statement 2 is not correct.
• In contrast to the Tibetans, the Andeans counter having less oxygen in every breath by having higher
hemoglobin concentrations in their blood. Haemoglobin is the protein in red blood cells that ferries
oxygen through the blood system. Having more haemoglobin to carry oxygen through the blood system
than people at sea level counterbalances the effects of hypoxia. Hence statement 1 is correct.

Q 53.B
An ecological niche is the role and position a species has in its environment; how it meets its needs for
food and shelter, how it survives, and how it reproduces. A species' niche includes all of its interactions
with the biotic and abiotic factors of its environment. Biotic factors are living things, while abiotic
factors are nonliving things. It is advantageous for a species to occupy a unique niche in an ecosystem
because it reduces the amount of competition for resources that species will encounter.

Q 54.A
• Phenotypic plasticity is the ability of an organism to change in response to stimuli or inputs from
the environment. Synonyms are phenotypic responsiveness, flexibility, and condition sensitivity. The
response may or may not be adaptive, and it may involve a change in morphology, physiological
state, or behaviour, or some combination of these, at any level of organization, the phenotype being
all of the characteristics of an organism other than its genes.
• The phenotype is the physical expression of the interaction between the genotype of an organism and its
environment. The phenotypes show variations due to differences in the environmental conditions within
the local conditions of the habitat.
• Such variation among individuals produced by the influence of the local conditions of the habitat is
known as phenotypic plasticity. Usually, species having a wide range of distribution evolve genetically
adapted populations called ecotypes.

Q 55.B
• Ecosystem services are the direct and indirect contributions of ecosystems to human well-being. They
support directly or indirectly our survival and quality of life. According to The Economics of Ecosystems
and Biodiversity (TEEB), ecosystem services can be categorized in four main types:
17 www.visionias.in ©Vision IAS

All Exam Study Material Available On - PHOTOCOPYNOTES.COM


Join Our Telegram Group - t.me/pdf4me
• Provisioning services are the products or primary product obtained from ecosystems such as food,
fresh water, wood, fiber, genetic resources and medicines.
• Regulating services are defined as the benefits obtained from the regulation of ecosystem processes such
as climate regulation, natural hazard regulation, water purification and waste management,
pollination or pest control.
• Habitat services highlight the importance of ecosystems to provide habitat for migratory species and to
maintain the viability of gene-pools.
• Cultural services include non-material benefits that people obtain from ecosystems such as spiritual
enrichment, intellectual development, recreation and aesthetic values.

• Hence option (b) is the correct answer.

Q 56.A
• A comprehensive milk safety and quality survey has questioned the perception of large-scale milk
adulteration in India. It was undertaken on 6,432 samples collected last year between May and October,
and picked from over 1,100 town/cities with over 50,000 population. The survey by an independent
agency at the behest of the Food Safety and Standards Authority of India (FSSAI) found 93% of the
samples were absolutely safe.
• The samples were tested for 13 common adulterants and three contaminants — pesticides, aflatoxin M1
and antibiotics. Only 12 adulterated samples were found to be unsafe for consumption. The adulterated
samples — they were also subjected to confirmatory tests — were from just three States: Telangana
(nine), Madhya Pradesh (two) and Kerala (one).
• The survey claims that quantitative analysis of all adulterated samples showed the amount of adulterants
and contaminants in the dozen samples was not high and hence “unlikely to pose serious threat” to human
health. However, it did find 368 samples (5.7%) had aflatoxin M1 residues beyond the permissible limit
of 0.5 microgram per kilogram.
• At 227, aflatoxin M1 was more widely present in processed milk samples than in raw milk (141). This is
the first time the presence of the contaminant in milk has been assessed.
• According to the FSSAI, aflatoxin M1 in milk is from feed and fodder, which is not regulated. The
highest residue levels of aflatoxin M1 in milk were seen in samples from three States — Tamil Nadu
(88 out of 551 samples), Delhi (38 out of 262) and Kerala (37 out of 187). According to the
International Agency for Research on Cancer the contaminant has been classified as “possibly
carcinogenic to humans”. Its carcinogenic potency is estimated to be about a one-tenth of aflatoxin B1.
• Since the current survey has limited itself to milk, it is not clear how widespread aflatoxin M1
contamination is in milk products such as cheese, and hence the total exposure to it.
• Aflatoxin M1 in milk and milk products is a public health concern especially in infants and young
children as milk constitutes one of the major sources of nutrients.
• According to the World Health Organisation, exposure to aflatoxin M1 in milk and milk products is
especially high in areas where the grain quality used as animal feed is poor. Hence all attempts need to
taken both before and after food crop harvest to reduce the toxin amount. Improper storage of food harvest
in warm and humid conditions leads to aflatoxin contamination that is much higher than what is seen in
the field. Equally important is in having facilities to regularly test for aflatoxin M1.

18 www.visionias.in ©Vision IAS

All Exam Study Material Available On - PHOTOCOPYNOTES.COM


Join Our Telegram Group - t.me/pdf4me
Q 57.C
Arrokoth is one of the thousands of known small icy worlds in the Kuiper Belt, the vast “third
zone” of the solar system beyond the inner terrestrial planets and the outer gas giant
planets. Arrokoth is a Native American term meaning “sky” in the Powhatan/Algonquian
language. Hence option (c) is the correct answer.
• Data from the newly named Arrokoth, has given clues about the formation of planets and our cosmic
origins.
• It was discovered in 2014 by a New Horizons team using the powerful Hubble Space Telescope.
• NASA launched the New Horizons mission in January 2006. After crossing by Pluto in 2015, in
January 2019 it flew by Arrokoth which was then provisionally named as Ultima Thule. This remains
the farthest flyby ever conducted.
• It was in the news, as recently it has been named as Arrokoth (previously known as Ultima
Thule)

Q 58.D
• Tropical forests are closed canopy forests growing within 28 degrees north or south of the
equator. They are very wet places, receiving more than 200 cm rainfall per year, either seasonally or
throughout the year. Rainforest trees are quite different from trees of temperate forests. In the
rainforest, trees grow to gigantic size, supported by strong, strut-like buttresses at the base of the trunk
that help to stabilize them in shallow forest soils. Huge creepers twine themselves around the trunks of
trees.
• Most tropical rainforest soils are relatively poor in nutrients. Millions of years of weathering and
torrential rains have washed most of the nutrients out of the soil. Tropical rain forest soils contain less
organic matter than temperate forests and most of the available nutrients are found in the living plant
and animal material. Nutrients in the soil are often in forms that are not accessible by plants. Constant
warmth and moisture promote rapid decay of organic matter. When a tree dies in the rainforest,
living organisms quickly absorb the nutrients before they have a chance to be washed away. When
tropical forests are cut and burned, heavy rains can quickly wash the released nutrients away, leaving the
soil even more impoverished.
• Few pure stands of trees exist in the rain forest. Though the tropics have great potential in timber
resources, commercial extraction is difficult. The trees do not occur in homogenous stands, there are no
frozen surfaces to facilitate logging and the tropical hardwoods are sometimes too heavy to float in the
rivers even if these flow in the desired directions.
• Hence option (d) is the correct answer.

Q 59.D
• The X-57, also known by nickname Maxwell, is an experimental aircraft being developed by National
Aeronautics and Space Administration (NASA). NASA showcased an early version of its first all-electric
experimental aircraft, the X-57 "Maxwell," at its lesser-known aeronautics lab in the California
desert. The research on X-57 plane began as part of NASA’s scalable convergent electric propulsion
operations research (SCEPTOR) project. The plane’s design was initiated under NASA’s leading-edge
asynchronous propeller technology.
• The Stratospheric Observatory for Infrared Astronomy (SOFIA) is an 80/20 joint project of NASA
and the German Aerospace Center (DLR) to construct and maintain an airborne observatory. NASA
awarded the contract for the development of the aircraft, operation of the observatory and management of
the American part of the project to the Universities Space Research Association (USRA) in 1996. The
DSI (Deutsches SOFIA Institute) manages the German parts of the project which are primarily science
and telescope related. SOFIA's telescope saw first light on May 26, 2010. SOFIA is the successor to the
Kuiper Airborne Observatory. It will observe celestial magnetic fields, star-forming regions, comets,
nebulae, and the galactic centre.

Q 60.D
• Commensalism
o This is the interaction in which one species benefits and the other is neither harmed nor
benefited. An orchid growing as an epiphyte on a mango branch, and barnacles growing on the
back of a whale benefit while neither the mango tree nor the whale derives any apparent
benefit.
o An orchid derives two basic benefits from this commensalistic relationship. As filter feeders, they
depend on the availability of plankton, which they filter into their bodies through feather-like
19 www.visionias.in ©Vision IAS

All Exam Study Material Available On - PHOTOCOPYNOTES.COM


Join Our Telegram Group - t.me/pdf4me
appendages extended through holes in their shells. When the whales swim into plankton-rich waters to
feed, so do the barnacles. They are consistently carried from feeding to feeding. Protection from
predators is another benefit. Barnacles attached to stationary objects often fall prey to fish, sea worms,
starfish and snails.
o The cattle egret and grazing cattle in the close association is a classic example of commensalism. The
egrets always forage close to where the cattle are grazing because the cattle, as they move, stir up and
flush out from the vegetation insects that otherwise might be difficult for the egrets to find and catch.
o Hence option (d) is the correct answer.
• Amensalism
o Amensalism is a type of biological interaction. It is an association between two organisms if different
species where one is adversely affected and the other stays unaffected.
• Parasitism
o Parasitism is a type of symbiotic relationship, or long-term relationship between two species, where
one member, the parasite, gains benefits that come at the expense of the host member.
• Mutualism
o This interaction confers benefits on both the interacting species. Lichens represent an intimate
mutualistic relationship between a fungus and photosynthesising algae or cyanobacteria.
o Similarly, the mycorrhizae are associations between fungi and the roots of higher plants. The fungi
help the plant in the absorption of essential nutrients from the soil while the plant in turn provides the
fungi with energy-yielding carbohydrates.
Q 61.A
• Radioactive materials are those materials or elements that emit radiation, thus they are not stable and get
transformed into other radioactive or non-radioactive materials. The harm that they can cause depends
on the radioactive elements and their half time function (the time needed for their concentration to
be reduced to half due to radioactive decay processes). Basically, the higher the half-time, the lower
the effects on human health. Radioactive elements with a short and very short half-time pose a serious
threat to human health because of their hazardous effects. Most of the radioactive materials have half-lives
of hundreds of thousands of years and, once generated, may persist in the environment for a very long
time.
• Cosmic Rays come from outer space to our planet with intense radiation as their nature, therefore,
causing radioactive pollution. Gamma rays, for example, are said to have the highest level of radiation
and yet, depending on their intensity, some are not visible to the human eye. The quantity with which the
rays hit the earth depends on the altitude of the earth and the geographical location.
• Radio isotopes are used to make detectors and in other industrial activities. Isotopes such as uranium have
high concentrations of radiation in them. On the other hand, common Isotopes such as carbon-containing
radioactive material are easily found in waterways through sewage lines. Examples of radioactive
isotopes includes Chromium-51, Cobalt-57 & 60, Calcium-47, Iodine-123, Krypton-85, Nickel-63
etc.
Q 62.D
• The most common hazard in forests is forests fire. During summer, when there is no rain for months, the
forests become littered with dry senescent leaves and twinges, which could burst into flames ignited by
the slightest spark. They pose a threat not only to the forest wealth but also to the entire regime to fauna
and flora seriously disturbing the bio-diversity and the ecology and environment of a region.
• Following are the environmental impacts of forest fires:
o The most noticeable impact of wildfires is stormwater runoff (storm water is water that originates
from precipitation events, including snow and ice melt). After the loss of vegetation, the ground’s
soil becomes hydrophobic and prevents the absorption of water. This inability to absorb water
promotes the transportation of debris and sediment into larger bodies of water, further polluting
valuable and essential resources. Post-fire flash floods become a threat and allow the introduction of
heavy metals from ash and soil to infiltrate waterways. Hence statement 1 is correct.
o As a forest burns, large amounts of smoke are released into the atmosphere. These smoke particles are
typically small and made up of gases and water vapor. Air pollution from fires have the potential to
travel great distances and oftentimes may pose a threat to human health.
o Forest fires affect the species population and distribution.
• Although wildfires leave an immense amount of destruction in their path, they do leave behind some
beneficial qualities too.
o Fire often clears out any invasive weeds, insects and disease that may have been affecting a
particular forest site, providing a chance for the area to have a fresh start with native species. Hence
statement 3 is correct.
20 www.visionias.in ©Vision IAS

All Exam Study Material Available On - PHOTOCOPYNOTES.COM


Join Our Telegram Group - t.me/pdf4me
o Fires also open the forest canopies to allow sunlight to reach the forest floor, benefitting the many
plants that are shade intolerant and cannot compete with more shade tolerant plants.
o Fires are a great way of clearing out the clutter. They can break down nutrients and minerals in
burning plants and other debris such as old logs, leaves and dense undergrowth and restore them to
the soil, thus making for a more fertile area. The increase in available nutrients in the soil after the
fire also helps create the perfect condition to boost microbial life in the forest floor. Hence statement
2 is not correct.
• Hence option (d) is the correct answer.

Q 63.C
• Phazolicin is an antibiotic produced by a soil bacterium, found in a tropical forest in
Mexico. Recently, discovered by an international team of scientists, it may help to tackle bacterial
infections in both plants and humans. The bacterium species belonging to a class of microbes called
Rhizobium,, present in the roots of plants. Phazolicin will prevent other, potentially harmful bacteria from
growing in the root system of agriculturally important plant. Hence, the bacterium can be used as a ‘plant
probiotic’. Hence option (c) is the correct answer.
• Phazolicin was produced by an unidentified species belonging to a class of microbes called Rhizobium,
present in the roots of plants. The study noted that the microbe was found in a tropical forest in Los
Tuxtlas, Mexico, in the soil and roots of wild beans called Phaseolus vulgaris

Q 64.D
• The Council of Scientific and Industrial Research (CSIR) recently announced the conclusion of a six-
month exercise of conducting a “whole-genome sequence” of 1,008 Indians. The project is part of a
program called “IndiGen”. The IndiGen initiative was undertaken by CSIR in April 2019, which was
implemented by the CSIR-Institute of Genomics and Integrative Biology (IGIB), Delhi and CSIR-Centre
for Cellular and Molecular Biology (CCMB), Hyderabad. CSIR first sequenced an Indian genome in
2009.
• A genome is the DNA, or sequence of genes, in a cell. Most of the DNA is in the nucleus and intricately
coiled into a structure called the chromosome. The rest is in the mitochondria, the cell’s powerhouse.
Every human cell contains a pair of chromosomes, each of which has three billion base pairs or one of
four molecules that pair in precise ways. The order of base pairs and varying lengths of these sequences
constitute the “genes”, which are responsible for making amino acids, proteins and, thereby, everything
that is necessary for the body to function. It is when these genes are altered or mutated that proteins
sometimes do not function as intended, leading to disease.
• Sequencing a genome means deciphering the exact order of base pairs in an individual. This
“deciphering” or reading of the genome is what sequencing is all about.
• Whole-genome data will be important for building the knowhow, baseline data and indigenous capacity in
the emerging area of Precision Medicine. Application of Genome Sequencing:
o The outcomes of the IndiGen will have applications in a number of areas including predictive and
preventive medicine with a faster and efficient diagnosis of rare genetic diseases.
o To find out any genetic abnormality in people.
o Help in identifying mutations responsible for cancer and develop a measure for it.
o Help us know why certain people do not react to certain drugs or have adverse reactions to some
drugs. For instance, some people less responsive to clopidogrel, a key drug that prevents strokes and
heart attacks.
o It will also lead to precision medication, instead of clinicians giving drugs based on collective
knowledge.

Q 65.A
• When wood is processed into paper, it produces a high-energy, toxic substance called black liquor.
Until the 1930s, black liquor from paper mills was considered a waste product and dumped into
nearby water sources.
• However, black liquor retains more than 50% of the wood’s biomass energy. With the invention of the
recovery boiler in the 1930s, black liquor could be recycled and used to power the mill. In the U.S., paper
mills use nearly all their black liquor to run their mills, and the forest industry is one of the most energy-
efficient in the nation as a result.
• More recently, Sweden has experimented in gasifying black liquor to produce syngas, which can then be
used to generate electricity.

21 www.visionias.in ©Vision IAS

All Exam Study Material Available On - PHOTOCOPYNOTES.COM


Join Our Telegram Group - t.me/pdf4me
Q 66.D
• National Aeronautics and Space Administration (NASA) has launched a satellite ICON to detect dynamic
zones of Earth’s Ionosphere. The satellite Ionosphere Connection Explorer (ICON) was launched from an
aircraft over the Atlantic Ocean near the Florida coast. The Ionospheric Connection Explorer will study
the frontier of space: the dynamic zone high in our atmosphere where Earth weather and space
weather meet. Here, the tenuous gases are anything but quiet, as a mix of neutral and charged
particles swirl in giant winds. Hence statement (d) is the correct answer.
• The ionosphere is the charged part of the upper atmosphere extending several hundred miles (kilometres)
up. It’s in constant flux as space weather bombards it from above and Earth weather from below,
sometimes disrupting radio communications.
• The refrigerator-size ICON satellite will study the airglow formed from gases in the ionosphere and
also measure the charged environment right around the spacecraft which is at a level of 580
kilometres above the Earth’s surface.
• A NASA satellite launched last year, Gold, is also studying the upper atmosphere.

Q 67.D
• Statement 1 is correct. Fly ash is a fine by-product recovered from gases of burning coal in thermal
power plants. They are micron sized earth elements primarily consisting silica, alumina and iron. The Fly
ash causes air pollution. It can also contaminate water and soil systems.
• Statement 2 is correct. The wet disposal of Fly ash results in leaching of toxic heavy metals in ground
water system.
• Fly ash can be used as a replacement for some of the Portland cement contents of concrete. It can be used
in the production of bricks for building construction. Central Government has made it mandatory for use
of fly ash bricks in construction activities happening 500km around thermal power plants.
• Statement 3 is correct. Maharashtra became the first state in the country to adopt the Fly Ash
Utilization Policy. The policy seeks 100% use of fly ash generated from thermal power plants and biogas
plants for construction activities.

Q 68.B
• Roots of the rubber tree - Ficus Elastica are the pavements for the natural root bridges in the thick forests
of Khasi Hills district. These types of bridges take very long to develop into a firm bridge. Normally it
takes around 10 to15 years, which is the normal construction period. These bridges grow to approximately
50 to100 feet long and have strong and deep roots providing a stable foothold. They can carry up to 50
people at a time. The life of these bioengineering wonders is estimated at 500 years, more than the life
span of most of man made modern bridges.
• The most famous Umshiang Double Decker root bridge in Nongriat village, south of Cherrapunjee about
70 km from the capital Shillong is estimated to be two centuries old.

Q 69.D
• Pollen preserves best if the sedimentary environment lacks oxygen or is acidic, conditions unfavorable
for the organisms that decompose pollen. Fossil pollen is an important kind of data for reconstructing past
vegetation. Because vegetation is sensitive to climate, fossil pollen is a very important kind of proxy data
for reconstructing past climates.
• Dendroclimatology is the science of determining past climates from trees (primarily properties of the
annual tree rings).
• Caves are underground chambers contain the secrets of Earth’s climate in what you may know as
stalactites, stalagmites, and other formations. Collectively, scientists call the icicle-shaped rocks hanging
from the ceiling and the rock mounds that rise up out of a cave’s floor “speleothems.” And, it is how these
speleothems form that allows them to hold a natural record of climate.
• Lacustrine deposits are sedimentary rock formations which formed in the bottom of ancient lakes. A
common characteristic of lacustrine deposits is that a river or stream channel has carried sediment into the
basin.
• Hence the correct answer is option (d).

Q 70.A
• Statement 1 is correct: FASTag is a device that employs Radio Frequency Identification (RFID)
technology for making toll payments directly from the prepaid or savings account linked to it. It is affixed
on the windscreen of the vehicle and enables the users to drive through toll plazas without stopping for
cash transactions.
22 www.visionias.in ©Vision IAS

All Exam Study Material Available On - PHOTOCOPYNOTES.COM


Join Our Telegram Group - t.me/pdf4me
• Benefits of using FASTag:
o Cashless payment – FASTag users don’t need to carry cash for toll transactions, and don’t need to
worry about collecting the exact change
o Promotional cashback – All FASTag users get a monthly cashback of 10% of toll transactions till 31
March 2017
o Faster transit – Auto-debit of exact amount through the FASTag enables faster transit through the toll
plazas and saves time
o Online recharge – FASTag can be recharged online through Credit Card / Debit Card / NEFT/ RTGS
or Net banking
o SMS alerts – Instant SMS alerts on a registered mobile number for toll transactions, low balance, etc.
• Statement 2 is not correct: FASTags are issued by 22 certified banks through various channels such as
Point-of-Sale (POS) at National Highway toll plazas and select bank branches. To ensure the easier
availability of FASTags, Indian Highways Management Company Ltd. (IHMCL), a company promoted
by NHAI, is signed MoUs with state-run Oil Marketing Companies (IOCL, BPC, and HPC). It will ensure
the availability of FASTags at petroleum outlets across India. IHMCL has been mandated to implement
the National Electronic Toll Collection Program (NETC) in the country.
• FASTags are part of the National Electronic Toll Collection (NETC) program developed by the National
Payments Corporation of India (NPCI).

Q 71.A
• Recently, the eighth session of the Governing Body of International Treaty of Plant Genetic Resources for
Food and Agriculture (ITPGRFA) held at Rome, Italy.
• The International Treaty on Plant Genetic Resources for Food and Agriculture was adopted by the Thirty-
First Session of the Conference of the Food and Agriculture Organization of the United Nations on 3
November 2001.
• ITPGRFA also is known as the Seed Treaty, is a comprehensive international agreement for
ensuring food security through the conservation, exchange and sustainable use of the world's plant
genetic resources for food and agriculture (PGRFA), as well as the fair and equitable benefit sharing
arising from its use. It also recognizes farmers' rights, subject to national laws.
• The Treaty’s truly innovative solution to access and benefit-sharing, the Multilateral System, puts 64
of our most important crops – crops that together account for 80 percent of the food we derive from plants
– into an easily accessible global pool of genetic resources that are freely available to potential users in the
Treaty’s ratifying nations for some uses.
• Statement 1 is correct: The Treaty facilitates access to the genetic materials of the 64 crops in the
Multilateral System for research, breeding, and training for food and agriculture. Those who access the
materials must be from the Treaty’s ratifying nations and they must agree to use the materials totally for
research, breeding, and training for food and agriculture.
• The Treaty prevents the recipients of genetic resources from claiming intellectual property rights over
those resources in the form in which they received them and ensures that access to genetic resources
already protected by international property rights is consistent with international and national laws.
• Statement 2 is correct: The Treaty recognizes the enormous contribution farmers have made to the
ongoing development of the world’s wealth of plant genetic resources. It calls for protecting the
traditional knowledge of these farmers, increasing their participation in national decision-making
processes and ensuring that they share in the benefits from the use of these resources.
• Statement 3 is not correct: The treaty creates a legally binding obligation on states that have joined it.
India passed Protection of Plant Varieties and Farmers’ Rights (PPV&FR) Act” to protect Farmers’ Rights
and breeder’s rights. Under act farmer is entitled to save, use, sow, resow, exchange, share or sell his farm
produce including seed of a variety protected under the PPV&FR Act, 2001 except brand name, and it
is legislation is fully compliant to article 9 of the Treaty.

Q 72.D
• Recently, The Pension Fund Regulatory and Development Authority (PFRDA) has permitted Overseas
Citizen of India to enrol in the National Pension System at par with NRIs.
• Statement 1 is correct: National Pension System (NPS) is a pension cum investment scheme launched by
the Government of India to provide old age security to citizens of India. A subscriber can contribute
regularly in a pension account during her working life, withdraw a part of the corpus in a lump sum and
use the remaining corpus to buy an annuity to secure a regular income after retirement.
• Statement 2 is not correct: Any individual citizen of India (both resident and Non-resident) in the age
group of 18-65 years (as on the date of submission of NPS application) can join NPS. Contributions made
23 www.visionias.in ©Vision IAS

All Exam Study Material Available On - PHOTOCOPYNOTES.COM


Join Our Telegram Group - t.me/pdf4me
by NRI are subject to regulatory requirements as prescribed by RBI and FEMA from time to time.
Overseas Citizens of India (OCI) can also enroll in National Pension System at par with NRIs.
• OCI willing to subscribe to NPS should be eligible to invest as per the provisions of the PFRDA Act and
the annuity/accumulated savings will be repatriable, subject to FEMA (Foreign Exchange Management
Act) guidelines.
• Statement 3 is not correct: The Scheme is regulated by Pension Fund Regulatory and Development
Authority (PFRDA). National Pension System Trust (NPST) established by PFRDA is the registered
owner of all assets under NPS. The Pension Fund Regulatory & Development Authority Act was passed
on 19th September, 2013 and the same was notified on 1st February, 2014. PFRDA is regulating NPS,
subscribed by employees of Govt. of India, State Governments and by employees of private
institutions/organizations & unorganized sectors. The PFRDA is ensuring the orderly growth and
development of pension market.

Q 73.B
• Below the tall trees in a tropical rain forest, a class of plants that live on the decaying tissues of the dead
plants and leaves survive. These are called saprophytes.
• There is another class of giant climbers that use the tall trees to climb up in search of sunlight. These are
called epiphytes.
• Xerophytes on the other hand are plants which are adapted to arid and semi arid conditions like cacti,
baobab etc.

Q 74.B
• Recently, an instrument on Chandrayaan-2, CLASS, has detected charged particles on Moon’s soil,
during the orbiter’s passage through the “Geotail”. The Geotail is a region in space that allows the
best observations. The region exists as a result of the interactions between the Sun and Earth.
Hence statement (b) is the correct answer.
• The Sun emits the solar wind, which is a continuous stream of charged particles. These particles are
embedded in the extended magnetic field of the Sun. Since the Earth has a magnetic field, it obstructs the
solar wind plasma.
• This interaction results in the formation of a magnetic envelope around Earth. On the Earth side facing the
Sun, the envelope is compressed into a region that is approximately three to four times the Earth radius.
• On the opposite side, the envelope is stretched into a long tail, which extends beyond the orbit of the
Moon. It is this tail that is called the Geotail. Once every 29 days, Moon traverses the Geotail for about 6
days centered around full moon.

Q 75.C
• The ozone depletion potential (ODP) of a chemical compound is the relative amount of degradation to
the ozone layer it can cause, with trichlorofluoromethane (CFC-11) being fixed at an ODP of 1.0.
• ODP can be estimated from the molecular structure of a given substance. Chlorofluorocarbons have
ODPs roughly equal to 1. Brominated substances have usually higher ODPs in range 5–15, because
of more aggressive bromine reaction with ozone. Hydrochlorofluorocarbons have ODPs mostly in
range 0.005 - 0.2 due to the presence of the hydrogen which causes them to react readily in the
troposphere, therefore reducing their chance to reach the stratosphere where the ozone layer is
present. Hydrofluorocarbons (HFC) have no chlorine content, so their ODP is essentially zero.

24 www.visionias.in ©Vision IAS

All Exam Study Material Available On - PHOTOCOPYNOTES.COM


Join Our Telegram Group - t.me/pdf4me
• Ozone depleting substances include:
o Chlorofluorocarbons (CFCs)
o Hydrochlorofluorocarbons (HCFCs)
o hydrobromoflurocarbons (HBFCs)
o halons (any of a number of unreactive gaseous compounds of carbon with bromine and other
halogens, used in fire extinguishers, etc.)
o methyl bromide
o carbon tetrachloride
o methyl chloroform.

• HFC, Ammonia and Carbon dioxide do not cause depletion of Ozone. So they have zero ODP.
Q 76.C
• Statement 1 is not correct: Littoral zone is the shallow water zone of the lake which supports rooted
vegetation.
• Statement 2 is not correct: Limnetic zone is the ideal location for the growth of phytoplanktons.
Profundal zone is the zone which does not receive any light.
• Statement 3 is correct: Benthic zone is the deepest layer among all the zones of lakes/water bodies, they
are habitat for benthic organisms like snail etc.

Q 77.C
• According to the source of emission pollutants are classified into two main groups: Primary and
Secondary pollutants.
• A primary pollutant is an air pollutant emitted from a source directly into the atmosphere. The source
can be either a natural process such as sandstorms and volcanic eruptions or anthropogenic (influenced by
humans) such as industrial and vehicle emissions.
• The major primary pollutants are Oxides of Sulphur, Oxides of Nitrogen, Oxides of Carbon, Particulate
Matter, Methane, Ammonia, Chlorofluorocarbons etc.
• A secondary pollutant is an air pollutant formed in the atmosphere as a result of the chemical or the
physical interactions between the primary pollutants themselves or between the primary pollutants and
other atmospheric components.
• Most important secondary level Air Pollutants are Ground Level Ozone, photo chemical Smog and
POPs (Persistent Organic Pollutants), Peroxyacetyl Nitrate (PAN). PAN is formed by interaction of
nitrogen oxide and hydrocarbon

Q 78.D
• Recently, the Trader’s body Confederation of All India Traders (CAIT) requesting the Commerce
Ministry for implementation of a ‘minimum operating price’ in order to create an even level playing field
between online and offline retailers.
• Offline and small retailers have been hit hard by the pricing policies of e-commerce outlets and big
retailers. Small businesses accuse these large conglomerates of “predatory pricing”, which involves
selling your product at extremely low prices
25 www.visionias.in ©Vision IAS

All Exam Study Material Available On - PHOTOCOPYNOTES.COM


Join Our Telegram Group - t.me/pdf4me
• Statement 1 is correct: MOP is the actual price at which a product is made available to a retailer. It is the
lowest price at which a retailer can sell a product and is set by the brands or the manufacturers. It is
usually at a marginal discount to the maximum retail price (MRP) and is decided by the demand and
supply dynamics of a product.
• Minimum Operating Price’ (MOP) consisting of landing price, operational cost and reasonable profit
margin and below the MOP no product should be sold in the market.
• Statement 2 is not correct: Minimum Operating Price ( MOP ) is not printed on the product. It is the
maximum retail price (MRP) that is printed on the product. MRP is the maximum price at which a
packaged product can be sold to the final consumer. The MOP can be lower or the same as the MRP.
• Statement 3 is not correct: The MOP is set by the manufacturer or the brand, not by the Department of
Consumer Affairs. The MOP is either lower than or equal to the selling price set by the retailer. According
to the Department of Consumer Affairs’ Standards of Weights and Measures (Packaged Commodities)
Rules, 1977, “‘maximum price’ in relation to any commodity in packaged form shall include all taxes
local or otherwise, freight, transport charges, commission payable to dealers, and all charges towards
advertisement, delivery, packing forwarding and the like, as the case may be.”
Q 79.A
• Saptaparni is a medium-sized evergreen tree from the very moist forests of the Himalayas. The name
comes from two Sanskrit words, Sapta meaning seven, and parni meaning leaves. As the name suggests,
the leaves, most often, are found in bunches of seven around the stem. They are blunt, glossy, and create
starry symmetries. The leaves remain throughout the year with new flushes standing out against older
leaves in March and April and again during the rainy season. The tree’s fruit is bean-like and appears in
pairs.
• The tree’s natural range shows a broad belt in the sub-Himalayan tract east of the Yamuna and is
also found in the moist forests of the Western and Eastern Ghats in peninsular India. In addition to
this, it also grows naturally from Sri Lanka to Myanmar and South China, and from Malay Peninsula to
Australia.
• The bark is known as ditabark, used by Indians as a traditional medicine to treat diarrhoea,
dysentery, asthma, and a few types of fevers. It has also been used as an aphrodisiac. When damaged,
the bark lets out a sticky milky latex, which is also valued for its medicinal properties. This is used by
vaids in formulations, and not to be consumed as is.
• The Saptaparni is of great cultural significance in the intellectual circle, as traditionally its leaves
were awarded to scholars and teachers during convocation ceremonies by the Visva Bharati
University. This tradition was started by Rabindranath Tagore in Gurudeb University. Due to
environmental degradation, the practice has been reduced to handing over a single leaf to the Vice-
Chancellor of the University.
• The large branches provide favourable breeding sites for wild bees to make their hives and the
pollination of the tree is carried out by insects. When the tree flowers, a plethora of insects like butterflies,
bees and beetles are seen all over it. The seeds of the tree have a tuft of silky hairs at each end, and are
dispersed by the wind.

Q 80.D
• E-waste denotes waste electrical and electronic equipment, whole or in part or rejects from their
manufacturing and repair process which are intended to be discarded and broadly comprises of discarded
computer monitors, motherboards, mobile phones and chargers, compact discs, headphones, Cathode Ray
Tubes (CRT), Printed Circuit Boards (PCB), televisions and so on.
• Electronics are full of valuable materials, including copper, tin, iron, aluminum, fossil fuels,
titanium, gold, and silver. Many of the materials used in making these electronic devices can be
recovered, reused, and recycled—including plastics, metals, and glass. Recycling e-waste enables us
to recover various valuable metals and other materials from electronics, saving natural resources (energy),
reducing pollution, conserving landfill space, and creating jobs. According to the EPA, recycling one
million laptops can save the energy equivalent of electricity that can run 3,657 U.S. households for a
year. Recycling one million cell phones can also recover 75 pounds of gold, 772 pounds of silver,
35,274 pounds of copper, and 33 pounds of palladium. In a report, Apple revealed that it recovered
2,204 pounds of gold —worth $40 million—from recycled iPhones, Macs, and iPads in 2015.
• E-waste also contains plastic, up to nearly 25 per cent of its weight. Novel recovery and conversion of e-
waste plastics to value-added products have also been successfully developed. The MeitY has developed
affordable technologies to recycle valuable materials and plastics in an environmentally sound
manner, including two exclusive PCB recycling technologies, viz 1000 kg/ day capacity (~35 MT e-
waste) and 100kg/batch (~3.5MT e-waste) processes, with acceptable environmental norms.
26 www.visionias.in ©Vision IAS

All Exam Study Material Available On - PHOTOCOPYNOTES.COM


Join Our Telegram Group - t.me/pdf4me
Q 81.D
• Primary production is defined as the amount of biomass or organic matter produced per unit area over a
time period by plants during photosynthesis. It is expressed in terms of weight or energy. The rate of
biomass production is called productivity. It can be divided into gross primary productivity (GPP) and
net primary productivity (NPP).
• Gross primary productivity of an ecosystem is the rate of production of organic matter during
photosynthesis. A considerable amount of GPP is utilised by plants in respiration. Gross primary
productivity minus respiration losses (R), is the net primary productivity (NPP). Net primary productivity
is the available biomass for the consumption to heterotrophs (herbivores and decomposers). Secondary
productivity is defined as the rate of formation of new organic matter byconsumers.
• The net primary productivity graph of different ecosystems is given in the image below.

• Habitats associated with estuaries, such as salt marshes and mangrove forests are among the highest
productivity zones. One reason that estuaries are such productive ecosystems is that the water filtering
through them brings in nutrients from the surrounding watershed. A Primary producers largely contribute
to making estuaries some of the most productive ecosystems on the Earth. There is generally greater
productivity near the coasts than in the open ocean.
• Because of its barren landscape, desert ecosystem has a low primary productivity rate compared to
other biomes. Savanna and Tundra ecosystems have intermediate range of primary productivity.
• Hence the correct answer is option (d).

Q 82.C
• Recently, Deputy Chief Minister of Haryana has, requesting the Prime Minister to “undertake cloud
seeding plan to combat air pollution engulfing Delhi and NCR”.
• Cloud seeding is a kind of weather modification technology to create artificial rainfall. It works only when
there are enough pre-existing clouds in the atmosphere. Cloud seeding aims to facilitate and accelerate the
process by making available chemical ‘nuclei’ around which condensation can take place.
• The success rate of these experiments in inducing rains is about 60 to 70 per cent, depending on local
atmospheric conditions, the amount of moisture in the air and cloud characteristics.
• Statement 1 is correct: The cloud seeding can be used for weather modification such as a reduction
in the rain and fog dispersal.
• Statement 2 is correct: In cloud seeding potassium chloride or calcium chloride, silver iodide, dry ice
(solid carbon dioxide) are injected into clouds to induce artificial rain.
• In clouds having a base at warm temperatures (warmer than zero degrees centigrade), hygroscopic seeding
with salt or potassium chloride or calcium chloride is used. In clouds with colder than zero degree
centigrade temperatures, glaciogenic seeding, whereby ice forming particles such as silver iodide is used.
Both types of seeding are important for clouds over the Indian region.

Q 83.A
• Bioremediation is the use of microbial species to clean up soil and groundwater that has been
contaminated by discharged chemicals. Bioremediation stimulates the growth of specific microbes that
use the discharged chemical contaminants as a source of food and energy.

27 www.visionias.in ©Vision IAS

All Exam Study Material Available On - PHOTOCOPYNOTES.COM


Join Our Telegram Group - t.me/pdf4me
• The microorganisms may be indigenous to a contaminated area or they may be isolated from elsewhere
and brought to the contaminated site.
• The process of bioremediation can be monitored indirectly by measuring the Oxidation Reduction
Potential or redox in the soil and groundwater, together with pH, temperature, oxygen content, electron
acceptor/ donor concentration, and concentration of breakdown products (eg carbon dioxide)
Bioremediation strategies
a) In situ bioremediation techniques
b) Ex-situ bioremediation techniques
Insitu remediation techniques
• It involves the treatment of the contaminated material at the site.
• Bioventing: In this, the supply of air and nutrients through wells to contaminated soil to stimulate
the growth of indigenous bacteria. It is used for hydrocarbons and can be used where the
contamination is deep under the surface.
• Biosparging: Injection of air under pressure below the water table to increase groundwater oxygen
concentrations and enhance the rate of biological degradation of contaminants by naturally occurring
bacteria.
• Bioaugmentation: Micro-organism are imported to a contaminated site to enhance the degradation
process.
Exsitu bioremediation techniques
• It involves the removal of the contaminant materials to be treated elsewhere.
• Landfarming: It is an ex-situ waste treatment process that is performed in the upper soil zone or in
biotreatment cells. Contaminated soils, sediments, or sludges are transported to the land farming site,
incorporated into the soil surface and periodically turned over (tilled) to aerate the mixture. In this
environment, the micro-organisms present in the soil are more able to break down the contaminants in
the soil.
• Biopiles: It is a process where piles of soil are placed over the top of a bug vacuum pump. The
vacuum pump pulls air through the pile of soil to allow oxygen to get thought the sol to the mico-
organisms. Contaminants that may be turned into gas forms are easily controlled as they are simply
sucked with the air stream through the soil.
• Composting: It is a treatment where the waste is mixed in with straw, hay or corn cobs to maximize
the water levels and air levels which the micro-organisms have access to. Composting can be done
three ways:1. Static Pile - where a pile of compost is aerated using vaccums or blowers.2.
Mechanically Agitated in-vessel composting - where the compost is put into a treatment tank where it
is turned around and mixed so it can be aerated.3. Windrow Composting - where the compost is laid
out in long piles where it can be mixed by a tractor
• Bioreactors: It involves the processing of contaminated solid material (soil, sediment, sludge) or
water through an engineered containment system.
Q 84.B
• Recent context- The Group of 20 (G20) Ministerial Meeting on Energy Transitions and Global
Environment for Sustainable Growth adopted a framework for action on marine plastic litter.
• In a Communique, the G20 Environment Ministers call for action to tackle marine plastic litter and
microplastics, among other types of marine litter, and to address the adverse impacts on marine
ecosystems, livelihoods, industries and human health.
• The ‘G20 Implementation Framework for Actions on Marine Plastic Litter,’ builds on the G20
Action Plan on Marine Litter that was adopted at the G20 Summit in 2017, in Hamburg, Germany.
• The new action framework is anticipated to complement the UN Environment Programme’s (UNEP)
work on marine litter and single-use plastics. The Implementation Framework aims to facilitate further
actions on marine litter while taking into account national policies, approaches and circumstances. To
implement the action plan, the G20 will promote a comprehensive life-cycle approach to “urgently and
effectively” prevent and reduce plastic litter discharge into the ocean.
• The approach will focus on land-based sources in particular, and pursue action including environmentally
sound waste management, clean up of marine plastic litter, prevention and reduction of plastic waste
generation and littering. It will also promote sustainable consumption and production (SCP), including
resource efficiency, circular economy and sustainable materials management.
• The action plan further promotes deployment of innovative solutions, in cooperation with existing
international initiatives and fora, and international cooperation to enhance national capacities.
28 www.visionias.in ©Vision IAS

All Exam Study Material Available On - PHOTOCOPYNOTES.COM


Join Our Telegram Group - t.me/pdf4me
Q 85.D
• Mimicry: Two species resemble each other closely, one species, called the mimic, is palatable to its
predators, but resembles another species, called the model, which is distasteful to the predator.
• Colouration: Concealing form and colouration enables a species to avoid its natural predator. For
example the brightly coloured and highly poisonous dart frogs of tropical rain forests of South America
are easily recognised and avoided by the predators.
• Aestivation or summer hibernation is spending the dry-hot period in an inactive state. It is observed in
ground squirrels in south-west deserts.
• Migration: It involves long-distance or short-distance movement of animals from one region to another.
For example, in Africa wild beasts migrate long distances, following geographical pattern of seasonal
rainfall and availability of fresh vegetation.

Q 86.C
Option (c) is correct.
In primary succession in water, the pioneers are the small phytoplanktons, they are replaced with time by
rooted-submerged plants, rooted-floating angiosperms followed by free-floating plants, then reed-swamp,
marsh-meadow, scrub and finally the trees. The climax community is a forest. With time the water body is
converted into land.

Q 87.D
• Causes of biodiversity loss: The accelerated rates of species extinctions that the world is facing now are
largely due to human activities. There are four major causes (‘The Evil Quartet’ is the sobriquet used to
describe them).
1. Habitat loss and fragmentation: This is the most important cause of driving animals and plants to
extinction. The most dramatic examples of habitat loss come from tropical rain forests. Once covering
more than 14 percent of the earth’s land surface, these rain forests now cover no more than 6 percent.
They are being destroyed fast. Besides total loss, the degradation of many habitats by pollution also
threatens the survival of many species. When large habitats are broken up into small fragments due to
various human activities, mammals and birds requiring large territories and certain animals with
migratory habits are badly affected, leading to population declines.
2. Over-exploitation: Humans have always depended on nature for food and shelter, but when ‘need’
turns to ‘greed’, it leads to over-exploitation of natural resources. Many species extinctions in the last
500 years (Steller’s sea cow, passenger pigeon) were due to overexploitation by humans. Presently
many marine fish populations around the world are over-harvested, endangering the continued
existence of some commercially important species.
3. Alien species invasions: When alien species are introduced unintentionally or deliberately for
whatever purpose, some of them turn invasive, and cause decline or extinction of indigenous species.
The Nile perch introduced into Lake Victoria in east Africa led eventually to the extinction of an
29 www.visionias.in ©Vision IAS

All Exam Study Material Available On - PHOTOCOPYNOTES.COM


Join Our Telegram Group - t.me/pdf4me
ecologically unique assemblage of more than 200 species of cichlid fish in the lake. You must be
familiar with the environmental damage caused and the threat posed to our native species by invasive
weed species like carrot grass (Parthenium), Lantana and water hyacinth (Eicchornia). The recent
illegal introduction of the African catfish Clarias gariepinus for aquaculture purposes is posing a
threat to the indigenous catfishes in our rivers.
4. Co-extinctions: When a species becomes extinct, the plant and animal species associated with it in an
obligatory way also become extinct. When a host fish species becomes extinct, its unique assemblage
of parasites also meets the same fate. Another example is the case of a coevolved plant-pollinator
mutualism where extinction of one invariably leads to the extinction of the other.

Q 88.A
• Sea ranching or ocean ranching is an internationally accepted concept, where fish that are commercially
important and need replenishment are reared in a controlled laboratory environment and let into the sea
when they are of a certain size.
• Recently, in an effort to replenish natural fish stock in the sea off the coast of Tamil Nadu, the Fisheries
Department has planned to ranch seeds of black tiger shrimps, Indian prawn and cobia at a cost of ₹10.5
crore.

Q 89.C
• Recently a report was released, which is the third edition of an exercise conducted by the Central Water
Commission (CWC) from May 2014 to April 2018. Samples taken from two-thirds of the water quality
stations spanning India’s major rivers showed contamination by one or more heavy metals, exceeding safe
limits set by the Bureau of Indian Standards. Iron emerged as the most common contaminant with 156
of the sampled sites registering levels of the metal above safe limits. None of the sites registered
arsenic levels above the safe limit.
• Long-term exposure to the above-mentioned heavy metals may result in slowly progressing physical,
muscular, and neurological degenerative processes that mimic Alzheimer’s disease, Parkinson’s disease,
muscular dystrophy and multiple sclerosis.
• The heavy metals cadmium, lead and mercury are also common air pollutants, being emitted
mainly as a result of various industrial activities. Although the atmospheric levels are low, they
contribute to the deposition and build-up in soils. Heavy metals are persistent in the environment and
are subject to bioaccumulation in food-chains. The study spanned 67 rivers in 20 river basins. Lead,
cadmium, nickel, chromium and copper contamination were more common in non-monsoon periods while
iron, lead, chromium and copper exceeded ‘tolerance limits’ in monsoon periods most of the time.
“Arsenic and zinc are the two toxic metals whose concentration was always obtained within the
limits throughout the study period,” the report noted.

Q 90.D
• Bharat stage (BS) emission standards are emission standards instituted by Central Government to regulate
the output of air pollutants from internal combustion engine equipment, including motor vehicles.
• The standards and the timeline for implementation are set by Central Pollution Control Board (CPCB)
under Ministry of Environment & Forests and climate change.
• These standards are based on European regulations were first introduced in 2000.
• The Union Government in October 2016 had decided to skip one stage and migrate to BS-VI directly
from BS-IV from April 2020 to fight the growing pollution. Hence statement 1 is not correct
• The harmful emissions that are identified for regulations in different Bharat Stages (BS) are carbon
monoxide (CO), unburnt hydrocarbons (HC), Nitrogen Oxides (NOx) and Particulate matter (PM). Each
stage specifies a certain limit on the pollutants released, Higher the Bharat Stage goes lesser it emits
pollutants.
• BS-I, BS-II and BS-III stages were launched in 2000, 2005 and 2010 respectively.
• At present, all new vehicles being registered are BS- IV-emission compliant. By switching to BS-VI,
India will join league of US, Japan and European Union, which follow Euro Stage VI emission norms.
• BS-IV fuels contain 50 parts per million (ppm) sulphur, while BS-V and BS-VI grade fuel will have
10 ppm sulphur. It will also bring down NOx emissions from diesel cars by 68% and 25% from
petrol engine cars. Cancer causing particulate matter emissions from diesel engine cars will also come
down by a phenomenal 80%. Hence statement 2 is not correct.

30 www.visionias.in ©Vision IAS

All Exam Study Material Available On - PHOTOCOPYNOTES.COM


Join Our Telegram Group - t.me/pdf4me
Q 91.C
• Blue baby syndrome, also known as infant methemoglobinemia, is a condition where a baby's skin turns
blue. This occurs due to a decreased amount of hemoglobin in the baby's blood. The most common cause
of blue baby syndrome is water contaminated with nitrates. After a baby drinks formula made with
nitrate-rich water, the body converts the nitrates into nitrites. These nitrites bind to the hemoglobin in the
body, forming methemoglobin, which is unable to carry oxygen.
• Blackfoot disease (BFD) is a severe form of peripheral vascular disease (PVD), in which the blood
vessels in the lower limbs are severely damaged, resulting eventually in progressive gangrene. It is
associated with long-term exposure to arsenic. It occurs in West Bengal and neighbouring areas with
elevated arsenic in drinking water.
• Itai-itai disease is the name given to the mass cadmium poisoning of Toyama Prefecture, Japan,
starting around 1912. The term "itai-itai disease" was coined for the severe pains of people with the
condition felt in the spine and joints. Cadmium poisoning can also cause softening of the bones and
kidney failure.
• Minamata disease is a neurological syndrome caused by severe mercury poisoning. Signs and
symptoms include ataxia, numbness in the hands and feet, general muscle weakness, loss of peripheral
vision, and damage to hearing and speech. Minamata disease was first discovered in Minamata city in
Kumamoto prefecture, Japan, in 1956. It was caused by the release of methylmercury in the industrial
wastewater.

Q 92.D
• The oligotrophic soils contain low amounts of nutrients. These soils generally develop in old and
geologically stable areas, such as soils found in much of the tropical rain forest region. Due to intense
weathering and high rates of leaching, these soils have a poor nutrient retention capacity. In nutrient poor
soils, nutrient accumulation in vegetation is high.
• Plant adaptations to oligotrophic soils provide for greater and more efficient nutrient uptake, reduced
nutrient consumption, and efficient nutrient storage. Improvements in nutrient uptake are facilitated by
root adaptations such as nitrogen-fixing root nodules, mycorrhizae and cluster roots.
• Cluster Roots: Cluster roots, are plant roots that form clusters of closely spaced short lateral rootlets.
They may form a two- to five-centimetre-thick mat just beneath the leaf litter to enhance nutrient uptake.
• A Mycorrhiza is a symbiotic association composed of a fungus and roots of a vascular plant. It helps in
efficient absorption of nutrients e.g. phosphorus.
• Nitrogen-fixing root nodules: Root nodules occur on the roots of plants that associate with symbiotic
nitrogen-fixing bacteria.

Q 93.D
• About INDAIR
o The Council of Scientific and Industrial Research-National Environmental Engineering Research
Institute (CSIR-NEERI) has launched the country’s first interactive online repository, called IndAIR
or Indian Air quality Interactive Repository. The repository is one of few such facilities in the
world and has the maximum number of studies-262 concerning Delhi-NCR.
o The project aims to make air quality research and studies in an easily accessible web format for
the public, media, researchers and academics. Hence statement (d) is the correct answer.
o It is the first comprehensive effort to enlist existing Indian research and analysis on air pollution, its
causes and effects. IndAIR has archived scanned documents from pre-Internet times (1950-1999),
research articles, reports and case studies, and over 2,000 statutes to provide the history of air
pollution research and legislation in the country. It includes all major legislations in the country dating
back to 1905.
o IndAIR began to document important milestones in India to make them available to public, help
academicians and also enable policy-makers to frame legislations. The web facility would also be a
useful platform for scientific community to share its current work and exchange ideas.
• About CSIR- NEERI
o The CSIR-National Environmental Engineering Research Institute (CSIR-NEERI) is a research
institute created and funded by Government of India. It was established in Nagpur in 1958 with focus
on water supply, sewage disposal, communicable diseases and to some extent on industrial pollution
and occupational diseases found common in post-independent India.
o It is a pioneer laboratory in the field of environmental science and engineering and part of Council of
Scientific and Industrial Research (CSIR).

31 www.visionias.in ©Vision IAS

All Exam Study Material Available On - PHOTOCOPYNOTES.COM


Join Our Telegram Group - t.me/pdf4me
o It has five zonal laboratories at Chennai, Delhi, Hyderabad, Kolkata and Mumbai. NEERI falls
under the Ministry of Science and Technology (India) of central government.
o The NEERI is an important partner organisation in India's POPs (Persistent Organic Pollutants)
national implementation plan (NIP).

Q 94.B
• Umbrella species are species that are selected for conservation-related decisions because the
conservation and protection of these species indirectly affects the conservation and protection of other
species within their ecosystem. Umbrella species help in the selection of potential reserve locations, as
well as the determination of the composition of the reserve.
• Umbrella species usually have a large area requirement for which the conservation of the species
extends the protection to other species sharing the same habitat. An example of an umbrella
species would be a Tiger. Efforts have been made to save the populations of wild Tigers in order to save
other species that are present in the same ecosystems such as Leopards, Monkey, Hares, Boars, etc.
Umbrella species are representative of other species in their habitat since they are known species, and they
also determine the area of conservation. The protection extended to the other species by the presence of
the umbrella species is known as the umbrella effect. Hence option (b) is the correct answer.
• Keystone Species: A keystone species is a species which has a disproportionately large effect on its
natural environment relative to its abundance. Such species are described as playing a critical role in
maintaining the structure of an ecological community, affecting many other organisms in an ecosystem
and helping to determine the types and numbers of various other species in the community. Without
keystone species, the ecosystem would be dramatically different or cease to exist altogether. Some
keystone species, such as the wolf, are also apex predators.
• Flagship species: A flagship species is a species selected to act as an ambassador, icon or symbol for a
defined habitat, issue, campaign or environmental cause. By focusing on, and achieving conservation
of that species, the status of many other species which share its habitat – or are vulnerable to the same
threats - may also be improved. Flagship species are usually relatively large, and considered to be
'charismatic' in western cultures. Flagship species may or may not be keystone species and may or may
not be good indicators of biological process.
• Indicator species: An indicator species is a species or group of species chosen as an indicator of,
or proxy for, the state of an ecosystem or of a certain process within that ecosystem. Examples
include crayfish as indicators of freshwater quality; corals as indicators of marine processes such as
siltation, seawater rise and sea temperature fluctuation; peregrine falcons as an indicator of pesticide
loads; or native plants as indicators for the presence and impact of alien species.
Q 95.A
• A biome is an area of the planet that can be classified according to the plants and animals that live in
it. Temperature, soil, and the amount of light and water help determine what life exists in a
biome. Biomes are distinct biological communities that have formed in response to a shared physical
climate. Biome is a broader term than habitat.
• A biome is different from an ecosystem. An ecosystem is the interaction of living and nonliving things
in an environment. A biome is a specific geographic area notable for the species living there. A
biome can be made up of many ecosystems. For example, an aquatic biome can contain ecosystems such
as coral reefs and kelp forests. Major biomes include deserts, forests, grasslands, tundra, and several types
of aquatic environments.
• Each biome consists of many ecosystems whose communities have adapted to the small differences in
climate and the environment inside the biome.”

Q 96.C
Thermal Pollution
• Thermal pollution is defined as sudden increase or decrease in temperature of a natural body of water
which may be ocean, lake, river or pond by human influence. This normally occurs when a plant or
facility takes in water from a natural resource and puts it back with an altered temperature. Usually, these
facilities use it as a cooling method for their machinery or to help better produce their products.
Effects
Decrease in the DO (Dissolved Oxygen) Levels
• Coldwater contains more oxygen than hot water so increases in temperature also decrease the oxygen-
carrying capacity of water (leads to anaerobic conditions in water bodies). In addition, raising the
32 www.visionias.in ©Vision IAS

All Exam Study Material Available On - PHOTOCOPYNOTES.COM


Join Our Telegram Group - t.me/pdf4me
water temperature increases the decomposition rate of organic matter in water, which also depletes
dissolved oxygen. These decreases in the oxygen content of the water occur at the same time that the
metabolic rates of the aquatic organisms, which are dependent on sufficient oxygen supply, are
rising because of the increasing temperature. Hence only options 2 and 3 are correct.
• Ocean invertebrates, fish and marine reptiles are obligate poikilotherms -- their body temperatures depend
entirely on the surrounding water. Each species is adapted to a single range of temperatures and many
pass through several different life stages, each with an individual range of tolerance. The ocean's vast
populations of microbes, fungi and sea plants also rely on narrow temperature ranges for optimal growth.
• Thermal pollution often temporarily increases aquatic plant populations. Other life forms, including
microbes and animals, move into these regions to exploit the higher oxygen levels, but when
photosynthesis stops at night or upon the death of plants, dissolved oxygen levels plummet, leading to
massive animal die-offs. Excess heat can also cause unnaturally large microbial blooms, which kill
animals by depleting local oxygen or producing toxins.

Q 97.D
• In September 2019, the Supreme Court held that Non-Government Organisations (NGOs) receiving
substantial financing from the government are bound to give information to the public under the
RTI Act. The bench was dealing with an issue on whether NGOs substantially financed by the
government fall within the ambit of 'public authority' under provisions of the Right to Information Act,
2005.
• In November 2019, a five-member Constitution Bench of the Supreme Court ruled that the office of the
Chief Justice of India is a “public authority” under the RTI Act, as much as the apex court itself. It
now enables the disclosure of information such as the judges’ personal assets. The Bench unanimously
argued that the right to know under the RTI Act was not absolute and this had to be balanced with the
right of privacy of judges. But the key takeaway from the judgment is that disclosure of details of serving
judges’ personal assets was not a violation of their right to privacy.
• The Prime Minister’s Office duly complies with the provisions of Section 4(1)(b) of the Right to
Information Act, 2005 for suo motu disclosure of the information.
Q 98.C
• The Global Hunger Index (GHI) is a tool designed to comprehensively measure and track hunger at the
global, regional, and country levels. The Global Hunger Index is a peer-reviewed annual report, jointly
published by Concern Worldwide and Welthungerhilfe.
• India is ranked 102 of 117 countries in the Global Hunger Index 2019, behind its neighbours Nepal,
Pakistan and Bangladesh. In 2018, India was ranked 103 out of 119 countries.
• Its GHI score has also decelerated — from 38.9 in 2005 to 32 in 2010 and then from 32 to 30.3 between
2010 and 2019. The GHI score is calculated on four indicators — undernourishment; child wasting,
the share of children under the age of five who are wasted (that is, who have low weight for their height,
reflecting acute undernutrition); child stunting, children under the age of five who have low height for
their age, reflecting chronic undernutrition; and child mortality, the mortality rate of children under the
age of five.
• The share of wasting among children in India rose from 16.5% in the 2008-2012 period to 20.8% in 2014-
2018, according to the report.
• Just 9.6% of all children between 6 and 23 months of age are fed a “minimum acceptable diet”, it said.
“India’s child wasting rate is extremely high at 20.8%, the highest for any country in this report,” it said.
• Neighbouring countries like Nepal (73), Sri Lanka (66), Bangladesh (88), Myanmar (69) and Pakistan
(94) are also in the ‘serious’ hunger category, but have fared better at feeding its citizens than India,
according to the report. China (25) has moved to a ‘low’ severity category and Sri Lanka is in the
‘moderate’ severity category.
• Hence only statements 1 and 3 are correct.
Q 99.A
• The tundra is the coldest of the biomes. It also receives low amounts of precipitation, making the
tundra similar to a desert. Tundra is found in the regions just below the ice caps of the Arctic,
extending across North America, to Europe, and Siberia in Asia. Much of Alaska and about half of
Canada are in the tundra biome. Tundra is also found at the tops of very high mountains elsewhere in
the world. Temperatures are frequently extremely cold, but can get warm in the summers.
• Tundra winters are long, dark, and cold, with mean temperatures below 0°C for six to 10 months of the
year. The temperatures are so cold that there is a layer of permanently frozen ground below the surface,
called permafrost. This permafrost is a defining characteristic of the tundra biome.
33 www.visionias.in ©Vision IAS

All Exam Study Material Available On - PHOTOCOPYNOTES.COM


Join Our Telegram Group - t.me/pdf4me
• Vegetation in the tundra has adapted to the cold and the short growing season. Mosses, sedges, and
lichens are common, while few trees grow in the tundra. The trees that do manage to grow stay close
to the ground so they are insulated by snow during the cold winters.
• Precipitation in the tundra totals 150 to 250 mm a year, including melted snow. That's less than most of
the world's greatest deserts.
• Hence option a is the correct answer.

Q 100.B
• The upper stratosphere consists of the considerable amount of ozone (O3), which protects us from the
harmful ultraviolet (UV) radiations (λ 255 nm) coming from the sun. These radiations cause skin cancer
(melanoma) in humans. Therefore, it is important to maintain the ozone shield.
• Ozone in the stratosphere is a product of UV radiations acting on dioxygen (O2) molecules. The UV
radiations split apart molecular oxygen into free oxygen (O) atoms. These oxygen atoms combine with the
molecular oxygen to form ozone.
Effects of Depletion of the Ozone Layer
• UV radiation has been linked to the development of cataracts, a clouding of the eye’s lens.
• With the depletion of ozone layer, more UV radiation filters into troposphere. UV radiations lead to
ageing of skin, cataract, sunburn, skin cancer, killing of many phytoplanktons, damage to fish productivity
etc.
• It has also been reported that plant proteins get easily affected by UV radiations which leads to the
harmful mutation of cells.
• It also increases evaporation of surface water through the stomata of the leaves and decreases the
moisture content of the soil. Increase in UV radiations damage paints and fibres, causing them to
fade.
• Increases in UV radiation could affect terrestrial and aquatic biogeochemical cycles, thus altering both
sources and sinks of greenhouse and chemically important trace gases (e.g., carbon dioxide, carbon
monoxide, carbonyl sulfide, ozone, and possibly other gases). These potential changes would contribute to
biosphere-atmosphere feedbacks that mitigate or amplify the atmospheric concentrations of these gases.
• Synthetic polymers, naturally occurring biopolymers, as well as some other materials of commercial
interest are adversely affected by UV radiation

34 www.visionias.in ©Vision IAS

All Exam Study Material Available On - PHOTOCOPYNOTES.COM

You might also like